Đăng ký Đăng nhập

Tài liệu Bất đẳng thức hay võ bá cẩn

.PDF
58
484
128

Mô tả:

Voõ Quoác Baù Caån Võ Quốc Bá Cẩn An Inequality collection Let the solutions say your method! The second version Caàn Thô © 2009 c 2009 by Vo Quoc Ba Can. Copyright All rights reserved. No part of this book may be reproduced or distributed in any form or by any means, or stored in data base or a retrieval system, without the prior written the permission of the author. www.mathvn.com www.mathvn.com Lời cảm ơn Những bài bất đẳng thức từ các cuộc thi giải toán Quyển tuyển tập này chắc chắn sẽ không thể thực hiện được nếu không có sự đóng góp của những người bạn của tôi. Họ đã trực tiếp động viên tôi thực hiện, gửi cho tôi những bài toán hay giúp tôi có thể tuyển tập lại một cách tốt nhất có thể các bài toán bất đẳng thức. Xin được nêu ra đây những người bạn thân thiết đã giúp đỡ tôi rất nhiều trong quá trình thực hiện quyển tuyển tập này Bài O1. Giả sử a, b, c là các số thực không âm thỏa mãn a2 + b2 + c2 + abc = 4. Chứng minh rằng 0 ≤ ab + bc + ca − abc ≤ 2. (USAMO 2000) Lời giải 1 (V. Q. B. Cẩn). Bất đẳng thức bên trái là hiển nhiên, bởi vì từ giả thiết, ta suy ra có ít nhất một số trong ba số a, b, c không lớn hơn 1. Giả sử số đó là c, khi đó ta sẽ có 1. Nguyễn Văn Dũng - Giảng viên Học Viện Kỹ Thuật Quân Sự Hà Nội. 2. Trần Quang Hùng - Cao học toán trường Đại Học Khoa Học Tự Nhiên, ĐHQG Hà Nội. ab + bc + ca − abc = ab(1 − c) + c(a + b) ≥ 0. 3. Cao Minh Quang - Giáo viên trường THPT Chuyên Nguyễn Bỉnh Khiêm, Vĩnh Long. 4. Võ Thành Văn - Lớp 12 Toán, trường THPT Chuyên, ĐHKH Huế. 5. Nguyễn Mạnh Dũng - Lớp 12 Toán, khối Phổ Thông Chuyên Toán – Tin, trường ĐHKHTN, ĐHQH Hà Nội. 6. Trần Anh Tuấn - đang cập nhật thông tin. Bây giờ, ta sẽ chứng minh bất đẳng thức bên phải. Thay abc = 4 − (a2 + b2 + c2 ) vào, ta có thể viết lại bất đẳng thức này thành a2 + b2 + c2 + ab + bc + ca ≤ 6. Ta sẽ dùng phương pháp phản chứng để chứng minh bất đẳng thức này. Giả sử tồn tại một bộ số (a, b, c) gồm các số hạng không âm sao cho a2 + b2 + c2 + abc = 4 và a2 + b2 + c2 + ab + bc + ca > 6. Khi đó, ta sẽ có √ 6(a2 + b2 + c2 ) 6 6abc 4 = a2 + b2 + c2 + abc = + √ 6 6 6 √ 6(a2 + b2 + c2 ) 6 6abc > 2 + , a + b2 + c2 + ab + bc + ca (a2 + b2 + c2 + ab + bc + ca)3/2 suy ra √ 3 6abc 2(ab + bc + ca) − (a2 + b2 + c2 ) > √ . a2 + b2 + c2 + ab + bc + ca Mặt khác, áp dụng bất đẳng thức Schur bậc 4 (ở dạng phân thức), ta thấy 2(ab + bc + ca) − (a2 + b2 + c2 ) ≤ 6abc(a + b + c) , a2 + b2 + c2 + ab + bc + ca nên từ trên ta suy ra √ 3 6abc . >√ a2 + b2 + c2 + ab + bc + ca p √ Điều này chứng tỏ rằng abc > 0 và 2(a + b + c) > 3(a2 + b2 + c2 + ab + bc + ca). Điều này vô lí, bởi vì ta luôn có 6abc(a + b + c) a2 + b2 + c2 + ab + bc + ca 3(a2 + b2 + c2 + ab + bc + ca) − 2(a + b + c)2 = a2 + b2 + c2 − ab − bc − ca ≥ 0. Như vậy, không thể nào tồn tại các số a, b, c thỏa mãn giả thiết của đề bài sao cho a2 + b2 + c2 + ab + bc + ca > 6, hay nói một cách khác, với mọi a, b, c không âm sao cho a2 + b2 + c2 + abc = 4, ta phải có ab + bc + ca − abc ≤ 2. Bài toán được chứng minh xong. Dễ thấy bất đẳng thức bên trái đạt được dấu bằng khi (a, b, c) là một hoán vị của bộ số (2, 0, 0); và bấtđẳng thức bên phải đạt được dấu bằng khi (a, b, c) = (1, 1, 1) hoặc √ √  (a, b, c) là một hoán vị của bộ số 2, 2, 0 . www.mathvn.com www.mathvn.com Những bài bất đẳng thức từ các cuộc thi giải toán 5 Lời giải 2. Đây là một chứng minh rất hay và đặc sắc cho bất đẳng thức bên phải. Trong ba số a, b, c, luôn tồn tại ít nhất 2 số sao cho hiệu của chúng khi trừ cho 1 có cùng dấu với nhau. Không mất tính tổng quát, giả sử hai số đó là a và b, khi đó ta có c(a − 1)(b − 1) ≥ 0, suy ra abc ≥ ac + bc − c. Mặt khác, theo bất đẳng thức AM – GM thì 4 = a2 + b2 + c2 + abc ≥ 2ab + c2 + abc, suy ra ab ≤ 2 − c. Từ đây, ta thu được 6 Lời giải 2 (V. Q. B. Cẩn). Ta sẽ dùng phương pháp phản chứng. Giả sử rằng tồn tại các số dương a+b+c < 1, dẫn a, b, c sao cho ab + bc + ca + abc = 4 và a + b + c < ab + bc + ca. Khi đó, ta có ab+bc+ca đến 4 = (ab + bc + ca) · 1 + abc · 1  2  3 a+b+c a+b+c > (ab + bc + ca) · + abc · ab + bc + ca ab + bc + ca (a + b + c)2 abc(a + b + c)3 = + . ab + bc + ca (ab + bc + ca)3 ab + bc + ca − abc ≤ (2 − c) + bc + ca − (ac + bc − c) = 2. Lời giải 3 (V. Q. B. Cẩn). Xin được giới thiệu thêm cùng bạn đọc một chứng minh khác cho bất đẳng thức bên phải. Từ giả thiết, ta dễ dàng chứng minh được tồn tại các số không âm x, y, z sao cho (x + y)(y + z)(z + x) > 0 và a = √ 2x , b = √ 2y , c = √ 2z . Với phép đặt thuần (x+y)(x+z) (y+z)(y+x) Từ đây, ta tìm được (z+x)(z+y) 2(ab + bc + ca) − (a2 + b2 + c2 ) > nhất này, ta có thể đưa bài toán về chứng minh 2∑ cyc xy 4xyz p − ≤ 1. (x + y) (x + z)(y + z) (x + y)(y + z)(z + x)  1 1 + x+z y+z xy xy +∑ =∑ cyc (y + z)(y + x) cyc (x + y)(x + z) xy zx =∑ +∑ cyc (x + y)(x + z) cyc (x + y)(x + z) xy xy p ≤∑ cyc x + y cyc (x + y) (x + z)(y + z) =∑ cyc abc(a + b + c)3 . (ab + bc + ca)2 Nhưng mà theo bất đẳng thức Schur bậc 3 ở dạng phân thức thì 2(ab + bc + ca) − (a2 + b2 + c2 ) ≤ 9abc a+b+c . Điều này dẫn đến 9abc abc(a + b + c)3 > , a + b + c (ab + bc + ca)2 Áp dụng bất đẳng thức AM – GM, ta có 2∑ Let the solutions say your method - Võ Quốc Bá Cẩn  suy ra abc > 0 và 9(ab + bc + ca)2 > (a + b + c)4 (mâu thuẫn bởi vì ta luôn có (a + b + c)2 ≥ 3(ab + bc + ca) theo AM – GM). Bởi vậy, ta không thể có a + b + c < ab + bc + ca với mọi a, b, c > 0 thỏa mãn giả thiết của đề bài. Điều này chứng tỏ rằng a + b + c ≥ ab + bc + ca, đây chính là điều phải chứng minh. Lời giải 3 (V. Q. B. Cẩn). Ta sẽ sử dụng phương pháp dồn biến để chứng minh bất đẳng thức đã cho. Để ý rằng ngoài điểm đẳng thức là a = b = c = 1 thì bất đẳng thức đã cho còn có một điểm "nhạy cảm" là a = b → 2, c → 0 (cùng các hoán vị). Điều này gợi cho ta giả sử c = min{a, b, c} và dùng phép dồn biến để đưa hai biến a, b về bằng nhau và bằng một số t dương nào đó. Muốn vậy, việc trước tiên ta phải làm đó là đảm bảo giả thiết của bài toán, tức là bộ số (t,t, c) phải thỏa mãn t 2 + 2tc + t 2 c = ab + bc + ca + abc = 4. Vì ta cần dồn biến từ (a, b, c) về (t,t, c) nên ta phải chứng minh a + b + c − ab − bc − ca ≥ 2t + c − t 2 − 2tc, x(y + z) 4xyz = 1+ . (x + y)(x + z) (x + y)(y + z)(z + x) Vì thế bất đẳng thức trên là hiển nhiên đúng, và phép chứng minh của ta được hoàn tất. Bài O2. Cho a, b, c là các số thực dương thỏa mãn ab + bc + ca + abc = 4. Chứng minh rằng tương đương a + b + c ≥ ab + bc + ca. (a + b − 2t)(1 − c) + (t 2 − ab) ≥ 0. (Việt Nam, 1996) 2y 2x 2z Lời giải 1 (V. Q. B. Cẩn). Từ giả thiết, suy ra ta có thể đặt a = y+z , b = z+x và c = x+y với x, y, z là các số thực dương. Khi đó, bất đẳng thức cần chứng minh có thể được viết lại thành x y z 2xy 2yz 2zx + + ≥ + + . y + z z + x x + y (x + z)(y + z) (y + x)(z + x) (z + y)(x + y) Áp dụng bất đẳng thức AM – GM, ta có   1 1 xy xy V P ≤ ∑ xy + =∑ +∑ 2 2 (x + z)2 (y + z)2 cyc cyc (z + x) cyc (y + z) xy x zx =∑ +∑ =∑ = V T. 2 2 cyc (y + z) cyc (y + z) cyc y + z 2t + c − t 2 − 2tc = 2t + Phép chứng minh của ta được hoàn tất. Dễ thấy đẳng thức xảy ra khi và chỉ khi x = y = z, tức là a = b = c = 1. www.mathvn.com (∗) Mặt khác, từ cách chọn của t, ta có c(a + b − 2t) = (c + 1)(t 2 − ab). Ta sẽ chứng minh a + b − 2t và t 2 − ab là những số không âm. Thật vậy, giả sử a + b − 2t < 0, khi đó ta cũng có t 2 − ab < 0. Điều 2 này dẫn đến ab > t 2 > (a+b) ≥ ab (vô lí). Vì vậy, ta phải có a + b − 2t ≥ 0 và t 2 − ab ≥ 0. Ngoài ra, 4 từ giả thiết của c, dễ thấy c ≤ 1. Và như thế, bất đẳng thức (∗) là hiển nhiên đúng. Phép dồn biến đã được hoàn tất, công việc còn lại của ta chỉ là chứng minh 2t + c − t 2 − 2tc ≥ 0 với t 2 + 2tc + t 2 c = 4. Đây là một công việc rất đơn giản, bởi vì từ t 2 + 2tc + t 2 c = 4, ta tìm được c = 2−t t ≥ 0, dẫn đến 2−t (2 − t)(t − 1)2 − t 2 − 2(2 − t) = ≥ 0. t t Lời giải 4 (V. Q. B. Cẩn). Dễ thấy rằng trong ba số a, b, c có ít nhất hai số có hiệu khi trừ cho 1 là những số cùng dấu với nhau. Giả sử hai số đó là a, b, khi đó ta sẽ có c(a − 1)(b − 1) ≥ 0, dẫn đến abc ≥ ac + bc − c. Từ đây, ta thu được a + b + c + abc ≥ (a + b)(c + 1). www.mathvn.com Những bài bất đẳng thức từ các cuộc thi giải toán 7 Mặt khác, áp dụng bất đẳng thức AM – GM, ta lại có 4 = abc + c(a + b) + ab ≤ (a + b)2 4 8 Let the solutions say your method - Võ Quốc Bá Cẩn Chứng minh rằng · c + c(a + b) + (a + b)2 4 abcd ≥ 3. , (Latvia 2002) Lời giải 1 (V. Q. B. Cẩn). Áp dụng bất đẳng thức Cauchy Schwarz, ta có suy ra 2 c≥ 4 − (a+b) 4 (a+b)2 4 + (a + b) = 4 4 − (a + b) = − 1. a+b a+b 1= Cộng 1 vào hai vế của bất đẳng thức này rồi nhân cho a + b > 0, ta thu được ngay (a + b)(c + 1) ≥ 4. Do đó, kết hợp với trên, ta được a + b + c + abc ≥ (a + b)(c + 1) ≥ 4 = ab + bc + ca + abc, hay nói một cách khác a + b + c ≥ ab + bc + ca. = Từ đó suy ra 1 a4 1 1 1 1 + + + a4 + 1 b4 + 1 c4 + 1 d 4 + 1 1 a4 1 a4 +1 + (Việt Nam, 2009)   1 2 k + 2c ≥ Lời giải (V. Q. B. Cẩn). Đầu tiên, ta cho a = b = 1, bất đẳng thức đã cho trở thành k + 1+c  1 3 k + 2 , tương đương (c − 1)2 (4k2 c + 4k2 + 2k − 1) ≥ 0. 8 (c + 1)2 2a 2b 2c Thật vậy, đặt x = b+c , y = c+a , z = a+b thì hiển nhiên xy + yz + zx + xyz = 4 và bất đẳng thức trên được viết lại thành (2k + x)(2k + y)(2k + z) ≥ (2k + 1)3 . Bây giờ, áp dụng bất đẳng thức AM – GM, ta dễ thấy xyz ≤ 1. Từ đó, sử dụng kết quả bài O2, ta thu được +1 + 1 a2 1 c4 1 c4 +1 + 1 d4 1 d4 +1 + b12 + c12 + d12 2 ≥ 1 a2 1 a4 + b12 + c12 + d12 2 + b14 + c14 + d14 + 4 . , tức là 1 1 1 1 1 1 + + + + + . a2 b2 a2 c2 a2 d 2 b2 c2 b2 d 2 c2 d 2 6 Mà theo bất đẳng thức AM – GM thì a21b2 + a21c2 + a21d 2 + b21c2 + b21d 2 + c21d 2 ≥ abcd nên kết hợp với trên, ta dễ dàng suy ra được bất đẳng thức cần chứng minh. Đẳng thức xảy ra khi và chỉ khi a = b = √ c = d = 4 3. Lời giải 2. Đặt x = a41+1 , y = b41+1 , z = c41+1 và t = d 41+1 thì ta có x + y + z + t = 1 và a4 = 1−x y+z+t = , x x b4 = z+t +x , y c4 = t +x+y , z d4 = x+y+z . t Từ đó, để chứng minh bất đẳng thức abcd ≥ 3, ta thấy rằng ta chỉ cần chứng minh được 4k2 + 2k − 1 Đến đây, cho c → 0, ta thấy bất đẳng thức chỉ đúng nếu ≥ 0. Ta sẽ chứng minh rằng, nghiệm của bất phương trình này chính là tập hợp tất cả các giá trị của k thỏa mãn yêu cầu bài toán, tức là chứng minh với 4k2 + 2k − 1 ≥ 0 thì       a b c 1 3 k+ k+ k+ ≥ k+ . b+c c+a a+b 2 1 b4 + b14 + c14 + d14 + 4 ≥ 2≥ Bài O3. Với a, b, c là các số thực dương bất kì, hãy tìm tất cả các số thực k để cho bất đẳng thức sau đúng       a b c 1 3 k+ k+ k+ ≥ k+ . b+c c+a a+b 2 1 b4 y+z+t z+t +x t +x+y x+y+z · · · ≥ 81. x y z t Nhưng bất đẳng thức này hiển nhiên đúng bởi vì theo AM – GM, ta có √ √ √ √ y + z + t z + t + x t + x + y x + y + z 3 3 yzt 3 3 ztx 3 3 txy 3 3 xyz · · · ≥ · · · = 81. x y z t x y z t Phép chứng minh của ta được hoàn tất. Bài O5. Cho các số dương a, b, c thỏa mãn (2k + x)(2k + y)(2k + z) = 8k3 + 4k2 (x + y + z) + 2k(xy + yz + zx) + xyz 1 1 1 + + ≥ 1. a+b+1 b+c+1 c+a+1 ≥ 8k3 + 4k2 (xy + yz + zx) + 2k(xy + yz + zx) + xyz = 8k3 + (4k2 + 2k)(4 − xyz) + xyz Chứng minh rằng = 8k3 + 16k2 + 8k − (4k2 + 2k − 1)xyz a + b + c ≥ ab + bc + ca. ≥ 8k3 + 16k2 + 8k − (4k2 + 2k − 1) = (2k + 1)3 . Như vậy, phép chứng minh của ta đã được hoàn tất. Điều này cũng chứng tỏ rằng khẳng định của ta ở trên là đúng, tức là tập hợp tất cả các giá trị cần tìm của k chính là nghiệm của bất phương trình 4k2 + 2k − 1 ≥ 0. (Andrei Ciupan, Chọn đội tuyển Romania dự thi Junior BMO 2007) Lời giải 1 (Andrei Ciupan). Áp dụng bất đẳng thức Cauchy Schwarz, dễ thấy (a + b + 1)(a + b + c2 ) ≥ (a + b + c)2 . Từ đó dẫn đến 1≤ Bài O4. Cho a, b, c, d là các số thực dương thỏa mãn 1 1 1 a + b + c2 b + c + a2 c + a + b2 + + ≤ + + , a + b + 1 b + c + 1 c + a + 1 (a + b + c)2 (a + b + c)2 (a + b + c)2 suy ra 1 1 1 1 + + + = 1. a4 + 1 b4 + 1 c4 + 1 d 4 + 1 (a + b + c)2 ≤ 2(a + b + c) + a2 + b2 + c2 , www.mathvn.com www.mathvn.com Những bài bất đẳng thức từ các cuộc thi giải toán 9 10 Let the solutions say your method - Võ Quốc Bá Cẩn Bài O6. Cho n ≥ 2 là một số nguyên bất kì. Tìm hằng số C nhỏ nhất để bất đẳng thức sau tức là a + b + c ≥ ab + bc + ca. xi x j (xi2 + x2j ) ≤ C(x1 + x2 + · · · + xn )4 , ∑ Bất đẳng thức của ta được chứng minh xong. Đẳng thức xảy ra khi và chỉ khi a = b = c = 1. Lời giải 2 (Cezar Lupu). Từ giả thiết, sử dụng bất đẳng thức Cauchy Schwarz, ta có       1 1 1 2 ≥ 1− + 1− + 1− a+b+1 b+c+1 c+a+1 a+b b+c c+a = + + a+b+1 b+c+1 c+a+1 [(a + b) + (b + c) + (c + a)]2 ≥ (a + b)(a + b + 1) + (b + c)(b + c + 1) + (c + a)(c + a + 1) 2(a2 + b2 + c2 ) + 4(ab + bc + ca) = 2 . (a + b2 + c2 ) + (ab + bc + ca) + (a + b + c) 1≤i< j≤n luôn đúng với mọi số thực không âm x1 , x2 , . . . , xn . (IMO 1999) Lời giải (V. Q. B. Cẩn). Với n = 2, cho x1 = x2 = 1, ta dễ thấy C ≥ 81 . Xét trường hợp n ≥ 3, cho x1 = x2 = 1, x3 = · · · = xn = 0, ta cũng tìm được C ≥ 18 . Ta sẽ chứng minh rằng 18 cũng chính là giá trị nhỏ nhất của C để bất đẳng thức trên đúng, tức là ∑ 1≤i< j≤n 1 xi x j (xi2 + x2j ) ≤ (x1 + x2 + · · · + xn )4 . 8 Thật vậy, áp dụng bất đẳng thức AM – GM, ta có Từ đây, ta suy ra được ! 2 2 2 2 2 2 (a + b + c ) + (ab + bc + ca) + (a + b + c) ≥ (a + b + c ) + 2(ab + bc + ca), ∑ xi x j (xi2 + x2j ) ≤ 1≤i< j≤n tức là ∑ xi x j xi2 + x2j + 1≤i< j≤n ∑ xk2 ! = n ∑ 1≤i< j≤n k6=i,k6= j ! xi x j n ! ∑ xi2 i=1 ! 1 · 2 ∑ xi x j · ∑ xi2 2 1≤i< j≤n i=1  2 n 2 ∑ xi x j + ∑ xi2 1 1≤i< j≤n i=1   =1 ≤   2 2 8 = a + b + c ≥ ab + bc + ca. Đây chính là điều phải chứng minh. Lời giải 3 (V. Q. B. Cẩn). Ta sẽ dùng phương pháp phản chứng để chứng minh bất đẳng thức này. 1 1 1 Giả sử tồn tại các số dương a, b, c sao cho a+b+1 + b+c+1 + c+a+1 ≥ 1 và a + b + c < ab + bc + ca. ab+bc+ca Khi đó, ta có 1 < a+b+c , dẫn đến n !4 ∑ xi . i=1 ab+bc+ca 1 ab + bc + ca a+b+c < = . a + b + 1 a + b + ab+bc+ca (a + b)(a + b + c) + ab + bc + ca a+b+c Như thế, khẳng định của ta đã được chứng minh xong. Điều này cho phép ta đi đến kết luận hằng số C nhỏ nhất thỏa mãn yêu cầu của đề bài là Cmin = 18 . Và ta thu được Bài O7. Chứng minh rằng với mọi số thực dương a, b, c, x, y, z, bất đẳng thức sau luôn được thỏa mãn ab + bc + ca ∑ (a + b)(a + b + c) + ab + bc + ca > 1, cyc ax by cz (a + b + c)(x + y + z) + + ≤ . a+x b+y c+z a+b+c+x+y+z tương đương  1 > ∑ 1− cyc  2(ab + bc + ca) , (a + b)(a + b + c) + ab + bc + ca (KMO Weekend Program 2007) Lời giải 1 (V. Q. B. Cẩn). Bất đẳng thức cần chứng minh có thể được viết lại như sau       ax b+y by c+z cz a + b + c + x + y + z (a + b + c)(x + y + z) a+x − + − + − ≥ − , 4 a+x 4 b+y 4 c+z 4 a+b+c+x+y+z hay là a2 + ab + b2 . (a + b)(a + b + c) + ab + bc + ca cyc 1>∑ Tuy nhiên, theo các bất đẳng thức AM – GM và Cauchy Schwarz thì hay là (a − x)2 (b − y)2 (c − z)2 (a + b + c − x − y − z)2 + + ≥ . a+x b+y c+z a+b+c+x+y+z 3 (a + b)2 3(a + b + c)2 VP ≥ ∑ ≥ 4 cyc (a + b)(a + b + c) + ab + bc + ca ∑ [(a + b)(a + b + c) + ab + bc + ca] cyc Theo bất đẳng thức Cauchy Schwarz, ta dễ thấy 3(a + b + c)2 3(a + b + c)2 ≥ = 1 (mâu thuẫn). = 2(a + b + c)2 + 3(ab + bc + ca) 2(a + b + c)2 + (a + b + c)2 VT ≥ 1 1 1 Vì vậy, ta không thể có điều giả sử trên, tức là với mọi a, b, c dương thỏa mãn a+b+1 + b+c+1 + c+a+1 ≥ 1 thì bắt buộc ta phải có a + b + c ≥ ab + bc + ca. Phép chứng minh được hoàn tất. www.mathvn.com [(a − x) + (b − y) + (c − z)]2 = V P, (a + x) + (b + y) + (c + z) và như thế, bất đẳng thức của ta đã được chứng minh xong. www.mathvn.com Những bài bất đẳng thức từ các cuộc thi giải toán 11 Lời giải 2 (Sanghoon). Áp dụng bất bất đẳng thức Cauchy Schwarz, ta có 12 Let the solutions say your method - Võ Quốc Bá Cẩn Lời giải 2. Đặt x =  √ √ 2 [(a + b + c)2 x + (x + y + z)2 a](a + x) ≥ (a + b + c) xa + (x + y + z) ax a b và y = bc , ta có c y = , a x 2 = ax(a + b + c + x + y + z) , a+b x+1 = , b+c 1+y b+c 1+y = . a+b 1+x Do đó, bất đẳng thức cần chứng minh có thể được viết lại thành từ đó suy ra ax (a + b + c)2 x + (x + y + z)2 a ≤ . a+x (a + b + c + x + y + z)2 1 y x+1 y+1 + + 1, x+ + ≥ y x y+1 x+1 Bằng cách thiết lập hai bất đẳng thức tương tự cho hai biểu thức còn lại, ta thu được by cz ax + + ≤ a+x b+y c+z (a + b + c)2 (x + y + z) + (x + y + z)2 (a + b + c) (a + b + c + x + y + z)2 tương đương x3 y2 + x2 + x + y3 + y2 ≥ x2 y + 2xy2 + 2xy. (a + b + c)(x + y + z) = . a+b+c+x+y+z Theo bất đẳng thức AM – GM, ta có x 3 y2 + x ≥ x2 y, 2 Bài toán được chứng minh xong. Bài O8. Cho các số thực dương a, b, c. Chứng minh bất đẳng thức sau Bài O9. Chứng minh rằng với mọi số thực dương a, b, c, ta đều có  √ 2 a + b + c + 3 abc 1 1 1 1 + + + √ ≥ . a + b b + c c + a 2 3 abc (a + b)(b + c)(c + a) (Belarus 1998) Lời giải 1 (V. Q. B. Cẩn). Để ý rằng bất đẳng thức đã cho tương đương với     a+b b+c a b c + + − 3 ≥ (a + b + c) + −2 , (a + b + c) b c a b+c a+b (Titu Andreescu, MOSP 1999) Lời giải 1 (V. Q. B. Cẩn). Áp dụng bất đẳng thức Cauchy Schwarz, ta có √ 2 3 abc c2 a2 b2 VT = 2 + 2 + 2 + c (a + b) a (b + c) b (c + a) 2abc   √ 2 √ 2 a + b + c + 3 abc a + b + c + 3 abc ≥ 2 = = V P. c (a + b) + a2 (b + c) + b2 (c + a) + 2abc (a + b)(b + c)(c + a) và như thế, nó có thể được viết lại thành a2 b2 c2 ab bc ca (a + b + c)(a − c)2 + + + + + − 2(a + b + c) ≥ . b c a c a b (a + b)(b + c) ab c ca + bc a + b ≥ a + b + c. Vì thế, ta chỉ cần chứng minh được a2 b2 c2 (a + b + c)(a − c)2 + + − (a + b + c) ≥ , b c a (a + b)(b + c) Bài toán được chứng minh xong. Đẳng thức xảy ra khi và chỉ khi a = b = c. Lời giải 2 (V. Q. B. Cẩn). Nhân cả hai vế của bất đẳng thức đã cho với (a + b)(b + c)(c + a) > 0, ta có thể viết lại nó dưới dạng hay là (a − b)2 (b − c)2 (c − a)2 (a + b + c)(a − c)2 + + ≥ . b c a (a + b)(b + c) 2 2 và x2 + y2 ≥ 2xy nên bất đẳng thức trên hiển nhiên đúng. Bài toán được chứng minh xong. a b c a+b b+c + + ≥ + + 1. b c a b+c a+b Theo bất đẳng thức AM – GM, ta dễ thấy x3 y2 + x + y3 + y3 ≥ 2xy2 , 2 ∑(a + b)(a + c) + cyc 2 √ (a + b)(b + c)(c + a)  3 √ ≥ a + b + c + abc , 2 3 abc 2 + (b−c) ≥ (a−c) Áp dụng bất đẳng thức Cauchy Schwarz, ta có (a−b) b c b+c . Do đó, ta chỉ cần chứng minh được 1 1 a+b+c + ≥ b + c a (a + b)(b + c) là một bất đẳng thức hiển nhiên đúng bởi vì nó tương đương với hay là ab + bc + ca + √ √ (a + b)(b + c)(c + a) 3 3 √ ≥ 2 abc(a + b + c) + a2 b2 c2 . 2 3 abc √ 3 Vì ab + bc + ca ≥ 3 a2 b2 c2 (theo AM – GM) nên ta chỉ cần chứng minh được √ √ (a + b)(b + c)(c + a) 3 3 √ + 2 a2 b2 c2 ≥ 2 abc(a + b + c), 2 3 abc b(a + b + c) ≥ 0. a(a + b)(b + c) tương đương Phép chứng minh của ta được hoàn tất. Đẳng thức xảy ra khi và chỉ khi a = b = c. www.mathvn.com √ 3 (a + b)(b + c)(c + a) + 4abc ≥ 4 a2 b2 c2 (a + b + c). www.mathvn.com Những bài bất đẳng thức từ các cuộc thi giải toán 13 Để chứng minh bất đẳng thức này, ta sẽ giả sử a ≥ b ≥ c, và viết lại nó như sau i   h √ √ √ 3 3 3 (b + c) (a + b)(a + c) − 4 a2 b2 c2 ≥ 4 a2 b2 c2 a − abc , (a + 1)2 (b + 1)2 (c + 1)2 1 1 1 + + ≤ + (4a + 1) + + (4b + 1) + + (4c + 1) = 8. 2a2 + (1 − a)2 2b2 + (1 − b)2 2c2 + (1 − c)2 3 3 3     √ √ √ 3 3 3 (b + c) a2 + ab + bc + ca − 4 a2 b2 c2 ≥ 4 a2 b2 c2 a − abc . √ 3 Lại sử dụng đánh giá ab + bc + ca ≥ 3 a2 b2 c2 một lần nữa, ta thấy rằng bất đẳng thức trên được suy ra từ       √ √ √ √ √ 3 3 3 3 3 (b+c) a2 − a2 b2 c2 ≥ 4 a2 b2 c2 a − abc , tương đương (b + c) a + abc ≥ 4 a2 b2 c2 . Theo bất đẳng thức AM – GM, ta có q   √ √ √ √ 3 3 3 (b + c) a + abc ≥ 2 bc · 2 a abc = 4 a2 b2 c2 . Do đó, bất đẳng thức cuối hiển nhiên đúng và phép chứng minh của ta được hoàn tất. Bài O10. Giả sử a, b, c là các số thực dương bất kì. Chứng minh bất đẳng thức sau (2b + c + a)2 (2c + a + b)2 (2a + b + c)2 + + ≤ 8. 2a2 + (b + c)2 2b2 + (c + a)2 2c2 + (a + b)2 2 2(b + c − a)2 2a2 + (b + c)2 + 2(c + a − b)2 2b2 + (c + a)2 2(b+c−a)2 , 2a2 +(b+c)2 + (USAMO 2003) nên ta có thể viết lại bất đẳng thức 2(a + b − c)2 2c2 + (a + b)2 2a2 + (b + c)2 ≥ 2(b + c − a)2 ≥ 1. 2a2 + 2(b2 + c2 ) = (b + c − a)2 a2 + b2 + c2 Đó chính là điều phải chứng minh. Bài O11. Cho x1 , x2 , y1 , y2 , z1 , z2 là các số thực thỏa mãn x1 , x2 > 0, x1 y1 > z21 và x2 y2 > z22 . Chứng minh rằng 1 1 8 . + ≥ x1 y1 − z21 x2 y2 − z22 (x1 + x2 )(y1 + y2 ) − (z1 + z2 )2 (IMO 1968) Lời giải 1 (V. Q. B. Cẩn). Từ giả thiết, dễ thấy y1 , y2 là các số dương. Điều này cho phép ta sử dụng bất đẳng thức AM – GM như sau √ (x1 + x2 )(y1 + y2 ) = x1 y1 + x2 y2 + (x1 y2 + x2 y1 ) ≥ x1 y1 + x2 y2 + 2 x1 y1 x2 y2 . Từ đánh giá này, đặt x1 y1 − z21 = a > 0 và x2 y2 − z22 = b > 0, ta thu được √ (x1 + x2 )(y1 + y2 ) − (z1 + z2 )2 ≥ x1 y1 + x2 y2 + 2 x1 y1 x2 y2 − (z1 + z2 )2 q = (a + z21 ) + (b + z22 ) + 2 (a + z21 )(b + z22 ) − (z1 + z2 )2 √  ≥ (a + z21 ) + (b + z22 ) + 2 ab + z1 z2 − (z1 + z2 )2 √ √ 2 = a+ b . Do đó, để chứng minh bất đẳng thức đã cho, ta chỉ cần chứng minh được   √ √ 2 1 1 a+ b + ≥ 8 (hiển nhiên đúng theo AM – GM). a b Bài toán được chứng minh xong. Đẳng thức xảy ra khi và chỉ khi x1 = x2 , y1 = y2 và z1 = z2 . Lời giải 2 (V. Q. B. Cẩn). Áp dụng bất đẳng thức Cauchy Schwarz, ta có    2  √ √ z1 z2 2 z z2 (z1 + z2 )2 = x1 · √ + x2 · √ ≤ (x1 + x2 ) 1 + 2 , x1 x2 x1 x2 Mà theo bất đẳng thức Cauchy Schwarz thì 2(b + c − a)2 Let the solutions say your method - Võ Quốc Bá Cẩn ta thu được hay là (2a+b+c) Lời giải 1 (V. Q. B. Cẩn). Để ý rằng 3 − 2a 2 +(b+c)2 = cần chứng minh dưới dạng 14 . Do đó, ta chỉ cần chứng minh được suy ra (b + c − a)2 + (c + a − b)2 + (a + b − c)2 ≥ a2 + b2 + c2 . (b+c−a)2 +(c+a−b)2 Bất đẳng thức này được suy ra từ bất đẳng thức sau ≥ c2 (đúng theo Cauchy 2 Schwarz) và hai bất đẳng thức tương tự. Như vậy, bài toán của ta đã được chứng minh xong. Dễ thấy đẳng thức xảy ra khi và chỉ khi a = b = c. Lời giải 2. Bất đẳng thức đã cho là một bất đẳng thức thuần nhất bậc 0. Vì thế, ta có thể chuẩn hóa cho a + b + c = 1, khi đó, nó được viết lại thành (a + 1)2 (b + 1)2 (c + 1)2 + + ≤ 8. 2a2 + (1 − a)2 2b2 + (1 − b)2 2c2 + (1 − c)2   z2 z2 (x1 + x2 )(y1 + y2 ) − (z1 + z2 )2 ≥ (x1 + x2 ) y1 + y2 − 1 − 2 x1 x2   x1 y1 − z21 x2 y2 − z22 = (x1 + x2 ) + x1 x2 s √ (x1 y1 − z21 )(x2 y2 − z22 ) ≥ 2 x1 x2 · 2 x1 x2 q 2 = 4 (x1 y1 − z1 )(x2 y2 − z22 ). Mặt khác, theo bất đẳng thức AM – GM thì Bây giờ, sử dụng đánh giá sau 1 1 2 + ≥q . x1 y1 − z21 x2 y2 − z22 2 (x1 y1 − z1 )(x2 y2 − z22 ) (a + 1)2 1 2(4a + 1) 1 2(4a + 1) 1 2(4a + 1) = + = + ≤ + , 2a2 + (1 − a)2 3 9a2 − 6a + 3 3 (3a − 1)2 + 2 3 2 www.mathvn.com www.mathvn.com Những bài bất đẳng thức từ các cuộc thi giải toán 15 16 Let the solutions say your method - Võ Quốc Bá Cẩn Vì thế [(x1 + x2 )(y1 + y2 ) − (z1 + z2 )2 ]  1 1 + x1 y1 − z21 x2 y2 − z22  ≥ 8, ∑ |xi + x j | = 1≤i< j≤n ∑ |xi + x j | + 1≤i< j≤k ∑ |xi + x j | + ∑ |xi + x j | 1≤i≤k k+1≤i< j≤n k+1≤ j≤n tức là k 1 1 8 . + ≥ x1 y1 − z21 x2 y2 − z22 (x1 + x2 )(y1 + y2 ) − (z1 + z2 )2 = k ∑ |xi | + (n − k) i=1 Nhận xét. Hoàn toàn tương tự, ta có thể chứng minh được bất đẳng thức tổng quát hơn vẫn còn đúng Nếu x1 , x2 , . . . , xn , y1 , y2 , . . . , yn và z1 , z2 , . . . , zn (n ≥ 2) là các số thực sao cho xi > 0 và xi yi > z2i thì n 1 n3 ∑ xi yi − z2 ≥  n   n   n 2 . i i=1 ∑ xi ∑ yi − ∑ zi i=1 i=1 ♣ Bài O12. Chứng minh rằng với mọi số thực x1 , x2 , . . . , xn , bất đẳng thức sau luôn được thỏa mãn ∑ 1≤i< j≤n |xi + x j | ≥ n−2 n ∑ |xi |. 2 i=1 j=k+1 k |x j | + ∑ n ∑ |xi + x j | i=1 j=k+1 k n k n ≥ k ∑ |xi | + (n − k) ∑ |x j | + ∑ ∑ (xi + x j ) i=1 i=1 j=k+1 j=k+1 k n k n = k ∑ |xi | + (n − k) ∑ |x j | + ∑ (n − k)xi + ∑ |x j | i=1 i=1 j=k+1 j=k+1 " # k k n n ≥ k ∑ |xi | + (n − k) ∑ |x j | + ∑ (n − k)xi + ∑ |x j | i=1 i=1 j=k+1 j=k+1 k n k n = k ∑ |xi | + (n − k) ∑ |x j | + (n − k) ∑ |xi | − k ∑ |x j | . i=1 i=1 j=k+1 j=k+1 Bài toán của ta đã được chứng minh xong. i=1 n ∑ Nếu k = 1 hoặc k = n − 1 thì thực hiện tương tự, ta cũng có đánh giá như trên. Như vậy, ta cần chứng minh ! n−2 k n k n k n k ∑ |xi | + (n − k) ∑ |x j | + (n − k) ∑ |xi | − k ∑ |x j | ≥ ∑ |xi | + ∑ |x j | . 2 i=1 i=1 i=1 j=k+1 j=k+1 j=k+1 Đặt A = ∑ki=1 |xi | và B = ∑nj=k+1 |x j | thì bất đẳng thức này trở thành (Chọn đội tuyển Romania dự thi IMO 2006) Lời giải (V. Q. B. Cẩn). Với n = 2, bất đẳng thức là hiển nhiên. Với n = 3, bất đẳng thức đã cho trở thành 1 |x1 + x2 | + |x2 + x3 | + |x3 + x1 | ≥ (|x1 | + |x2 | + |x3 |). 2 Trong ba số x1 , x2 , x3 có ít nhất hai số cùng dấu với nhau, giả sử đó là x2 và x3 , khi đó ta có |x2 + x3 | = |x2 | + |x3 |, suy ra bất đẳng thức trên có thể được viết lại thành 1 1 |x1 + x2 | + |x1 + x3 | + |x2 + x3 | ≥ |x1 |. 2 2 kA + (n − k)B + |(n − k)A − kB| ≥ n−2 (A + B). 2 Nếu (n − k)A ≥ kB, ta có n−2 n−2 n+2 n + 2 − 4k A− B= A+ B 2 2 2 2 2 n+2 k n + 2 − 4k (n − 2k) + 2n ≥ · B+ B= B ≥ 0. 2 n−k 2 2(n − k) V T −V P = kA + (n − k)B + (n − k)A − kB − Nếu (n − k)A ≤ kB, ta có n−2 n−2 n+2 4k + 2 − 3n A− B= B+ A 2 2 2 2 2 n+2 n−k 4k + 2 − 3n (n − 2k) + 2n ≥ · A+ A= A ≥ 0. 2 k 2 2k Sử dụng bất đẳng thức trị tuyệt đối, ta có V T −V P = kA + (n − k)B − (n − k)A + kB − 1 1 |x1 + x2 | + |x1 + x3 | + |x2 + x3 | ≥ (|x1 + x2 | + |x1 + x3 | + |x2 + x3 |) 2 2 1 1 ≥ |(x1 + x2 ) + (x1 + x3 ) − (x2 + x3 )| = |x1 | ≥ |x1 |. 2 2 Bài toán được chứng minh xong. Vậy bất đẳng thức đã cho cũng đúng cho n = 3. Bây giờ ta xét trường hợp n ≥ 4. Rõ ràng nếu tất cả các số xi đều cùng dấu với nhau (tức là cùng âm hoặc cùng không âm) thì bất đẳng thức đã cho là hiển nhiên. Vì thế, trong chứng minh của ta, ta chỉ cần xét trường hợp thứ ba, tức là trong dãy xi tồn tại vừa số âm lẫn số không âm. Do vai trò ngang nhau giữa các biến nên không mất tính tổng quát, ta có thể giả sử x1 ≤ · · · ≤ xk ≤ 0 ≤ xk+1 ≤ · · · ≤ xn . Nếu 2 ≤ k ≤ n − 2 thì ta có www.mathvn.com Bài O13. Cho a, b, c là các số thực dương sao cho a ≤ b ≤ c và x, y, z là các số dương bất kì. Chứng minh rằng x y z (a + c)2 (x + y + z)2 ≥ (ax + by + cz) + + . 4ac a b c (Olympic toán Áo 1971) www.mathvn.com Những bài bất đẳng thức từ các cuộc thi giải toán 17 18 Let the solutions say your method - Võ Quốc Bá Cẩn Lời giải (V. Q. B. Cẩn). Áp dụng bất đẳng thức AM – GM, ta có (MOSP 2004) Lời giải (V. Q. B. Cẩn). Ta viết lại bất đẳng thức cần chứng minh dưới dạng sau h  x y z i2 y 4ac(ax + by + cz) + + + + . ≤ (ax + by + cz) + ac a b c a b c x z (cos x)cot x > sin x, Vì vậy, ta chỉ cần chứng minh được (a + c)(x + y + z) ≥ (ax + by + cz) + ac x a hay là (cos2 x)cot x > sin2 x.  Áp dụng bất đẳng thức Bernoulli với để ý rằng cot x > 1 ∀x ∈ 0, π4 , ta được z y + + , b c (cos2 x)cot x = (1 − sin x)cot x (1 + sin x)cot x ≥ (1 − sin x · cot x)(1 + sin x · cot x) = sin2 x. hay là  Đẳng thức xảy ra khi sin x = 0 hoặc cot x = 1, nhưng cả hai điều này là không thể xảy ra do x ∈ 0, π4 . Vì vậy, ta đi đến (cos2 x)cot x > sin2 x. y(a − b)(b − c) ≥ 0. b Bất đẳng thức này hiển nhiên đúng do a ≥ b ≥ c. Do đó, phép chứng minh của ta được hoàn tất. Đó chính là điều phải chứng minh. Bài O14. Cho n + 1 số thực x0 , x1 , . . . , xn thỏa mãn x0 = 0, xi ≥ 0 với mọi i = 1, 2, . . . , n và x1 + x2 + · · · + xn = 1. Chứng minh rằng n xi π √ 1≤∑√ ≤ . 2 i=1 1 + x0 + · · · + xi−1 xi + · · · + xn (Olympic toán Trung Quốc 1996) Lời giải. Đầu tiên, ta sẽ chứng minh vế bất đẳng thức bên trái. Theo bất đẳng thức AM − GM thì n n xi n xi ∑ √1 + x0 + · · · + xi−1 √xi + · · · + xn ≥ 2 ∑ 1 + x0 + · · · + xi−1 + xi + · · · + xn = ∑ xi = 1. i=1 Bài O16. Cho n ≥ 2 là một số nguyên dương cho trước và x1 , x2 , . . . , xn là các số thực dương bất kì. Đặt   1 1 1 Sn = min x1 , + x2 , . . . , + xn , . x1 xn−1 xn Hãy tìm giá trị lớn nhất của Sn theo n. (Tập huấn đội tuyển Việt Nam dự thi IMO 2009) π Lời giải (V. Q. B. Cẩn). Ta sẽ chứng minh rằng giá trị lớn nhất của Sn là 2 cos n+2 . Thật vậy, giả sử π Sn > 2 cos n+2 , khi đó ta có i=1 i=1   1 1 1 π + xn , > 2 cos . min x1 , + x2 , . . . , x1 xn−1 xn n+2 Vế bên trái được chứng minh xong. Bây giờ, ta sẽ đi đến chứng minh vế bên phải. Từ giả thiết cho phép ta đặt x0 + x1 + · · · + xi = sin αi 0 ≤ αi ≤ π2 với mọi i = 0, 1, . . . , n. Khi đó, dễ thấy 2 2 Đặt ai = 2 (1 + x0 + · · · + xi−1 )(xi + · · · + xn ) = 1 − (x0 + · · · + xi−1 ) = 1 − sin αi = cos αi , và như vậy, bất đẳng thức cần chứng minh có thể được viết lại thành n ∑ i=1 sin (i+1)π n+2 iπ sin n+2 với mọi i = 1, 2, . . . , n thì ta dễ thấy ai > 0 và a1 = sin αi − sin αi−1 π ≤ . cos αi−1 2 1 1 1 π + a2 = · · · = + an = = 2 cos . a1 an−1 an n+2 Bây giờ, ta sẽ chứng minh rằng xi > ai với mọi i = 1, 2, . . . , n. Khi đó, theo giả thiết phản chứng, ta sẽ có π 1 1 π < < = 2 cos . 2 cos n + 2 xn an n+2 Ta có đánh giá sau αi − αi−1 αi + αi−1 αi − αi−1 cos ≤ 2 sin cos αi−1 2 2 2 αi − αi−1 ≤ 2· · cos αi−1 = (αi − αi−1 ) cos αi−1 , 2 sin αi − sin αi−1 = 2 sin Đó là điều vô lí, và ta sẽ có ngay điều phải chứng minh. Để chứng minh khẳng định trên, ta hãy để ý rằng nếu có một số k (k ≤ n − 1) nào đó mà xk > ak thì 2 cos suy ra n ∑ i=1 n sin αi − sin αi−1 (αi − αi−1 ) cos αi−1 π ≤∑ = αn − α0 = αn ≤ . cos αi−1 cos αi−1 2 i=1 1 π 1 = + ak+1 > + ak+1 . n + 2 ak xk π Mà theo giả thiết phản chứng thì x1k + xk+1 > 2 cos n+2 nên kết hợp với trên, ta có ngay xk+1 > ak+1 . Điều này chứng tỏ rằng nếu khẳng định của ta đúng với k thì nó cũng đúng cho mọi i = k, k + 1, . . . , n. Nhưng rõ ràng x1 > a1 (theo giả thiết phản chứng) nên từ đó, ta suy ra được xi > ai với mọi i = π 1, 2, . . . , n. Từ chứng minh này, kết hợp với lập luận ở trên, ta thấy rằng đánh giá Sn > 2 cos n+2 là π không thể xảy ra, hay nói một cách khác, với mọi n ≥ 2 thì Sn ≤ 2 cos n+2 . Dễ thấy đẳng thức xảy ra được khi xi = ai nên đây cũng chính là giá trị lớn nhất của Sn . Bài toán được giải quyết xong. Bài toán được chứng minh xong. Bài O15. Chứng minh rằng với mọi 0 < x < π4 , bất đẳng thức sau luôn được thỏa mãn (cos x)cos x > (sin x)sin x . www.mathvn.com www.mathvn.com Những bài bất đẳng thức từ các cuộc thi giải toán 19 Bài O17. Cho a1 , a2 , . . . , an là các số thực thỏa mãn |ai | ≤ 1 với mọi i = 1, 2, . . . , n và a1 + a2 + · · · + an = 0. Chứng minh rằng tồn tại một số k ∈ {1, 2, . . . , n} sao cho 2k + 1 . |a1 + 2a2 + · · · + kak | ≤ 4 (Tập huấn đội tuyển Việt Nam dự thi IMO 2009) Lời giải (V. Q. B. Cẩn). Đặt b0 = 0, bi = a1 + · · · + iai với mọi i = 1, 2, . . . , n thì ta có ai = bi −bi i−1 với mọi i = 1, 2, . . . , n. Như vậy, từ giả thiết ta có |bi − bi−1 | ≤ i và n   b0 n−1 bi bi bn n−1 bi bn bi − bi−1 =− +∑ − + =∑ + . i 1 i i + 1 n n i=1 i=1 i=1 i(i + 1) k ≥ |bk − bk−1 | = |bk | + |bk−1 |. Nếu |bk | > Bài O19. Chứng minh rằng với mọi a, b, c, d dương, ta đều có a+c b+d c+a d +b + + + ≥ 4. a+b b+c c+d d +a (Dự tuyển IMO 1971) Lời giải. Áp dụng bất đẳng thức AM – GM, ta có   1 1 4(a + c) a+c c+a + = (a + c) + ≥ . a+b c+d a+b c+d a+b+c+d Hoàn toàn tương tự, ta cũng có Không mất tính tổng quát, ta có thể giả sử b1 > 0, bởi vì nếu b1 = 0 thì bài toán hiển nhiên được thỏa mãn, còn nếu b1 < 0 thì ta có thể thay ai bởi −ai , lúc này giả thiết của bài toán vẫn không đổi nhưng ta sẽ có b01 > 0. Bây giờ, từ giả thiết này, ta thấy rằng trong dãy b2 , . . . , bn tồn tại ít nhất một số không dương, ta gọi k là chỉ số nhỏ nhất sao cho bk ≤ 0, khi đó ta có bk−1 > 0, và 2k+1 4 Let the solutions say your method - Võ Quốc Bá Cẩn n 0 = ∑ ai = ∑ i=1 20 và |bk−1 | > 2(k−1)+1 4 |bk | + |bk−1 | > Vì vậy ta phải có |bk | ≤ 2k+1 4 b+d d +b 4(b + d) + ≥ . b+c d +a a+b+c+d Cộng tương ứng vế với vế hai bất đẳng thức này, ta dễ dàng thu được bất đẳng thức cần chứng minh. Đẳng thức xảy ra khi và chỉ khi a = c và b = d. Bài O20. Cho a, b, c là các số thực dương có tổng bằng 3. Chứng minh bất đẳng thức sau thì ta có b c 3 a + + ≥ . b + c2 c + a2 a + b2 2 2k + 1 2(k − 1) + 1 + = k (mâu thuẫn với trên). 4 4 hoặc |bk−1 | ≤ 2(k−1)+1 . 4 (Phạm Kim Hùng, Tập huấn đội tuyển Việt Nam dự thi IMO 2009) Lời giải (V. Q. B. Cẩn). Sử dụng bất đẳng thức Cauchy Schwarz, ta dễ thấy Bài toán được chứng minh xong.  √ √ √ 2 V T · [a2 (b + c2 ) + b2 (c + a2 ) + c2 (a + b2 )] ≥ a a + b b + c c . Bài O18. Cho u1 , u2 , . . . , un , v1 , v2 , . . . , vn là các số thực bất kì. Chứng minh rằng ! ! n n n 4 1 + ∑ (ui + vi )2 ≤ 1 + ∑ u2i 1 + ∑ v2i . 3 i=1 i=1 i=1 Bất đẳng thức cần chứng minh được đưa về  √ √ √ 2 2 a a + b b + c c ≥ 3(a2 b + b2 c + c2 a) + 3(a2 b2 + b2 c2 + c2 a2 ). (Dự tuyển IMO 1970) Lời giải (V. Q. B. Cẩn). Áp dụng bất đẳng thức Cauchy Schwarz, ta có v u n n n n n n u 2 2 2 2 2 ∑ (ui + vi ) = ∑ ui + ∑ vi + 2 ∑ ui vi ≤ ∑ ui + ∑ vi + 2t i=1 i=1 i=1 i=1 i=1 i=1 Nhân cả hai vế của bất đẳng thức này cho 3, rồi sử dụng các đánh giá sau n ! ∑ u2i n ∑ v2i i=1  √ √ √ √ 2 a2 b2 6 a a + b b + c c = 6 ∑ a3 + 12 ∑ ab ab ≥ 6 ∑ a3 + 24 ∑ a cyc cyc cyc cyc + b ! ! 2 2 a b (a + b + c) = 2 ∑ a3 ∑a +8∑ a+b cyc cyc cyc ! ! ab = 2 ∑ a3 ∑ a + 8 ∑ a2 b2 + 8abc ∑ a + b , cyc cyc cyc cyc ! . i=1 Vì vậy, để chứng minh bất đẳng thức đã cho, ta chỉ cần chứng minh được 4 (1 + a2 )(1 + b2 ) ≥ a2 + b2 + 2ab + 1, 3 q q trong đó a = ∑ni=1 u2i và b = ∑ni=1 v2i . Ta có 2 2 2 2 2 và 9(a2 b + b2 c + c2 a) = 3(a + b + c)(a2 b + b2 c + c2 a) = 3(a3 b + b3 c + c3 a) + 3 ∑ a2 b2 + 3abc ∑ a 2 4(1 + a )(1 + b ) − 3(a + b + 2ab + 1) = (a − b) + (2ab − 1) ≥ 0, cyc nên bất đẳng thức trên hiển nhiên đúng. Bài toán được chứng minh xong. Dễ thấy đẳng thức xảy ra khi và chỉ khi ui = vi và u21 + u22 + · · · + u2n = 12 . www.mathvn.com cyc ≤ (a2 + b2 + c2 )2 + 3 ∑ a2 b2 + 3abc ∑ a, cyc cyc www.mathvn.com Những bài bất đẳng thức từ các cuộc thi giải toán 21 22 Let the solutions say your method - Võ Quốc Bá Cẩn bc Đặt A = a2 − 1 + (a+b)(a+c) (dễ thấy A ≤ 0), bất đẳng thức cần chứng minh có thể được viết lại dưới dạng     aA a 1 1 + (b + c)2 ≤ . [(b + c)2 + Abc] bc + ≤ , tương đương a(b + c) + Ab2 c2 + bc b+c 4 b+c 4 ta có thể đưa bài toán về chứng minh bất đẳng thức mạnh hơn là ! ! ab 2 ∑ a3 ∑ a + 8 ∑ a2 b2 + 8abc ∑ a + b ≥ ∑ a4 + 14 ∑ a2 b2 + 3abc ∑ a, cyc cyc cyc cyc cyc cyc cyc tương đương ab ∑ a + 2 ∑ ab(a + b ) + abc ∑ a − 6 ∑ a b ≥ 4abc ∑ a − 2 ∑ a + b cyc cyc cyc cyc cyc cyc 4 2 2 2 2 Ta có 14 − a(b + c) = !  bc cyc  aA + (b + c)2 ≤ 2bc(b + c − a)2 , b+c Đến đây, ta đặt t = mạnh hơn là ∑ a4 + abc ∑ a ≥ ∑ ab(a2 + b2 ). cyc b+c 2 ≤ 3 ∑ ab(a2 + b2 ) − 6 ∑ a2 b2 ≥ 4abc cyc ab ∑a−2∑ a+b cyc ! thì dễ thấy hay là bc (a+b)(a+c) ≤ t2 (a+t)2 2(b + c − a)2 ≥ aA + (b + c)2 . b+c nên ta chỉ cần chứng minh bất đẳng thức   t2 a 2 a −1+ + 4t 2 . 2t (a + t)2 Thay a = 1 − 2t vào, bất đẳng thức này trở thành . cyc 2(4t − 1)2 ≥ x(b − c)2 + y(c − a)2 + Không mấy khó khăn, ta có thể dễ dàng viết lại bất đẳng thức này dưới dạng z(a − b)2 ≥ 0, trong đó x = 3bc − 2abc b+c và các biểu thức y, z tương tự. Không mất tính tổng quát, giả sử a ≥ b ≥ c, khi đó ta dễ thấy z ≥ y ≥ x, lại có x + y = 3ac + 3bc − 1 3 2(2t − a)2 ≥ Vì vậy, bất đẳng thức trên được suy ra từ cyc ≥ 2bc(b + c − a)2 và Ab2 c2 ≤ 0 nên bất đẳng thức này được suy ra từ . Theo bất đẳng thức Schur bậc 4 thì cyc (b+c−a)2 4   1 − 2t t2 (1 − 2t)2 − 1 + + 4t 2 , 2 2t (1 − t) tương đương 2(4t − 1)2 − 4t 2 ≥ (1 − 2t)(2t − 2) + 2abc 2abc 2abc 2abc − ≥ 3ac + 3bc − − = ac + bc > 0, b+c a+c b a t(1 − 2t) , 2(1 − t)2 hay là nên x + y > 0, từ đó ta suy ra được z ≥ y > 0. Đến đây, với chú ý rằng (a − c)2 ≥ (b − c)2 , ta có 2(16t 2 − 11t + 2) ≥ x(b − c)2 + y(c − a)2 + z(a − b)2 ≥ (x + y)(b − c)2 + z(a − b)2 ≥ 0. Bài toán được chứng minh xong. Dễ thấy đẳng thức xảy ra khi và chỉ khi a = b = c = 1. Bài O21. Cho a, b, c là các số thực không âm thỏa mãn không có hai số nào trong chúng đồng thời bằng 0 và a + b + c = 1. Chứng minh bất đẳng thức sau     a b c 1 bc + ca + ab + ≤ . b+c c+a a+b 4 (Tập huấn đội tuyển Việt Nam dự thi IMO 2009) Lời giải (V. Q. B. Cẩn). Không mất tính tổng quát, giả sử a = max{a, b, c}. Khi đó, ta có biến đổi sau    c c2 a b2 a bc b ab + = a2 bc + + + ca + c+a a+b a + b a + c (a + b)(a + c) bc2 b2 c bc = a2 bc + b2 + c2 − − + a + b a + c (a + b)(a + c)   c b (a + b + c)2 = (b + c)2 + bc a2 − 2 − − + a + b a + c (a + b)(a + c)   bc 2 2 = (b + c) + bc a − 1 + . (a + b)(a + c) www.mathvn.com t(1 − 2t) . 2(1 − t)2 2 2 2 Ta có 4(1 − t)2 ≥ 4 1 − 13 = 16 9 > 1 và 16t − 11t + 2 − t(1 − 2t) = 2(1 − 3t) ≥ 0 nên bất đẳng thức cuối hiển nhiên đúng. Bài toán được chứng minh xong. Dễ thấy đẳng thức xảy ra khi và chỉ khi a = b = 12 , c = 0 cùng các hoán vị tương ứng. Bài O22. Cho p, q là các số tự nhiên thỏa mãn q ≥ p. Xét n+1 (n ≥ 2) số thực a0 = 0, a1 , . . . , an−1 , an = 1 thỏa mãn ak−1 + ak+1 ak ≤ ∀k = 1, 2, . . . , n − 1. 2 Chứng minh rằng n−1 n−1 k=1 k=1 (p + 1) ∑ akp ≥ (q + 1) ∑ aqk . (Chọn đội tuyển Romania dự thi IMO 2006) Lời giải. Từ giả thiết, ta dễ thấy 0 = a0 ≤ a1 ≤ · · · ≤ an = 1, và 0 ≤ a1 = a1 − a0 ≤ a2 − a1 ≤ · · · ≤ an − an−1 = 1 − an−1 . Một nhận xét hữu ích giúp ta có thể đưa bài toán về trường hợp khá đơn giản, đó là ta chỉ cần chứng minh bất đẳng thức đã cho trong trường hợp q = p + 1 là đủ. Bây giờ, sử dụng công thức tổng Abel, ta có n n n n−1 k k=1 k=1 k=1 k=1 i=1 ∑ akp+1 = ∑ ak · akp = an ∑ akp − ∑ (ak+1 − ak ) ∑ aip . www.mathvn.com Những bài bất đẳng thức từ các cuộc thi giải toán 23 p+1 p Để ý rằng an = 1 nên ∑nk=1 akp+1 = ∑n−1 + 1 và an ∑nk=1 akp = ∑nk=1 akp = ∑n−1 k=1 ak k=1 ak + 1, vì thế n−1 n−1 n−1 k k=1 k=1 k=1 i=1 24 Let the solutions say your method - Võ Quốc Bá Cẩn Do đó, bất đẳng thức đã cho được suy ra từ (a − c)2 (b − d)2 (a + 2c)(a − c)(b − d) + ≥ . a+b+c b+c+d (a + b + c)(a + c + d) ∑ akp+1 = ∑ akp − ∑ (ak+1 − ak ) ∑ aip . Theo bất đẳng thức AM – GM, ta có Do ai − ai−1 ≤ ak+1 − ak với mọi i = 1, . . . , k, nên k k i=1 i=1 (b − d)2 2(a − c)(b − d) (a − c)2 + ≥p . a+b+c b+c+d (a + b + c)(b + c + d) (ak+1 − ak ) ∑ aip ≥ ∑ (ai − ai−1 )aip . Vì thế, ta chỉ cần chứng minh được p+1 ≥ (p + 1)ai−1 aip (theo bất đẳng thức AM – GM suy rộng), nên Lại có paip+1 + ai−1 (ai − ai−1 )aip ≥ p+1 aip+1 − ai−1 p+1 r 2(a + c + d) . Nếu a ≥ d thì ta có Từ những lập luận này, ta thu được d ≥ a thì ta dễ thấy n−1 n−1 n−1 k k=1 k=1 k=1 i=1 n−1 n−1 k = p+1 aip+1 − ai−1 p+1 k=1 i=1 ∑ akp − k=1 (p + 1) ∑ akp n−1 ≥ (p + 2) ∑ k=1 nên bất đẳng thức cuối hiển nhiên đúng. Bài toán được chứng minh xong. Dễ thấy đẳng thức xảy ra khi và chỉ khi a = c và b = d. akp+1 , k=1 hay nói một cách khác, bất đẳng thức đã cho đúng trong trường hợp q = p + 1. Vì vậy, phép chứng minh của ta được hoàn tất. Đẳng thức xảy ra khi và chỉ khi p = q, hoặc a1 = · · · = an−1 = 0, hoặc p = 0, q = 1 và ak = nk . Bài O23. Chứng minh rằng với mọi số thực dương a, b, c, d, ta đều có (a − b)(a − c) (b − c)(b − d) (c − d)(c − a) (d − a)(d − b) + + + ≥ 0. a+b+c b+c+d c+d +a d +a+b (Dự tuyển IMO 2008) Lời giải (V. Q. B. Cẩn). Đặt P(a, b, c, d) là vế trái của bất đẳng thức đã cho. Không mất tính tổng quát, ta thấy rằng ta có thể giả sử (a − c)(b − d) ≥ 0. Thật vậy, nếu (a − c)(b − d) ≤ 0, ta lấy a1 = b, b1 = c, c1 = d, d1 = a thì ta cũng có P(a1 , b1 , c1 , d1 ) = P(a, b, c, d), và lúc này ta lại có (a1 − c1 )(b1 − d1 ) = −(a − c)(b − d) ≥ 0. Bây giờ, ta hãy để ý rằng (a − b)(a − c) (c − d)(c − a) (a − c)2 (a + 2c)(a − c)(b − d) + = − , a+b+c c+d +a a + b + c (a + b + c)(a + c + d) và (b − d)2 ≥ 1 và 2(a + c + d) ≥ a + 2c nên bất đẳng thức này là hiển nhiên. Nếu q a+c c+d nên bất đẳng thức trên là hệ quả của ≥ p √ √ √ 2(a + c + d) a + c = 2 a + c + d (a + c + d)(a + c) ≥ 2(a + c) a + c + d √ √ ≥ 2(a + c) c + d ≥ (a + 2c) c + d, 1 n−1 p+1 ∑ ak . p + 1 k=1 Do đó n−1 a+b+c b+c+d Ta có ∑ akp − ∑ ∑ k=1 n−1 a+b+c √ √ 2(a + c + d) a + c ≥ (a + 2c) c + d. ∑ akp+1 ≤ ∑ akp − ∑ ∑ (ai − ai−1 )aip ≤ q q b+c+d a+b+c ≥ a + 2c. b+c+d Bài O24. Cho các số thực dương a, b, c, d thỏa mãn đồng thời hai điều kiện abcd = 1 và a+b+c+d > a b c d b + c + d + a . Chứng minh rằng b c d a + + + > a + b + c + d. a b c d (Dự tuyển IMO 2008) Lời giải (V. Q. B. Cẩn). Sử dụng các bất đẳng thức Cauchy Schwarz và AM – GM, ta có         a b c d b c d a 1 1 1 1 + + + + + + + = (a + c) + + (b + d) + b c d a a b c d b d a c   1 1 = (a + c)(b + d) + ac bd    1 1 1 1 1 √ +√ ≥ (a + c)(b + d) √ + √ 2 ac ac bd bd   2 2 2 1 ≥ (a + c)(b + d) + · q√ √ 2 a+c b+d ac bd = (a + b + c + d) + (a + b + c + d)   a b c d > + + + + (a + b + c + d). b c d a (b − d)2 (b − c)(b − d) (d − a)(d − b) (b + 2d)(a − c)(b − d) + = + ≥ . b+c+d d +a+b b + c + d (b + c + d)(d + a + b) b+c+d www.mathvn.com www.mathvn.com Những bài bất đẳng thức từ các cuộc thi giải toán 25 26 Let the solutions say your method - Võ Quốc Bá Cẩn Từ đó ta suy ra b c d a + + + > a + b + c + d. a b c d   √ √ √ √ t 2 2z 3 · 3x + 2 · 2y + 2k · + k · k k   4z2 t 2 2 2 2 ≤ (5k + 5) 3x + 2y + 2 + 2 , k k √ !2 √ √ √ √ 3z √ 2t + 2k · (6x + 4y + 3z + 2t)2 = 6 · 6x + 2 · 2y + 3k · k k   3z2 2t 2 ≤ (10 + 5k2 ) 6x2 + 4y2 + 2 + 2 , k k √ √ !2 √ √ √ 2z √ 3t (4x + 6y + 2z + 3t)2 = 2 · 2x + 6 · 6y + 2k · + 3k · k k   2z2 3t 2 ≤ (10 + 5k2 ) 4x2 + 6y2 + 2 + 2 , k k   √ √ √ √ z 2t 2 (2x + 3y + z + 4t)2 = 2 · 2x + 3 · 3y + k · + 2k · k k   z2 4t 2 2 2 2 ≤ (5k + 5) 2x + 3y + 2 + 2 . k k (3x + 2y + 4z + t)2 = Đó chính là điều phải chứng minh. Bài O25. Cho a, b, c, d là các số thực thỏa mãn điều kiện nhất và giá trị nhỏ nhất của biểu thức sau 1 2 ≤ a2 + b2 + c2 + d 2 ≤ 1. Tìm giá trị lớn Q = (a − 2b + c)2 + (b − 2c + d)2 + (b − 2a)2 + (c − 2d)2 . (Chọn đội tuyển Việt Nam q dự thi IMO 1993) Lời giải (V. Q. B. Cẩn). Trước hết ta sẽ tìm giá trị lớn nhất của Q. Đặt k = đẳng thức Cauchy Schwarz, ta có √ 5−1 2 , áp dụng bất  a2 2 2 + 2b + c , k k2  2   2 d d (b − 2c + d)2 = b + (−c) + (−c) + · k ≤ (k2 + 3) b2 + 2c2 + 2 , k k  2  a 2 a 2a 2 2 2 +b , (b − 2a) = · k + · k + (−b) ≤ (2k + 1) k k k2   2  2 d d 2d (c − 2d)2 = + c2 . · k + · k + (−c) ≤ (2k2 + 1) 2 k k k (a − 2b + c)2 = a · k + (−b) + (−b) + c 2 ≤ (k2 + 3)  Cộng tương ứng vế với vế các bất đẳng thức này, ta thu được   3 a2 + b2 + c2 + d 2 ≤ 25(3k2 + 5)(x2 + y2 ) + 25 2 + 2 (z2 + t 2 ). k Cộng tương ứng vế với vế các bất đẳng thức này, ta thu được q√ 5−1 Do k = nên 3k2 + 5 = 2 + k32 = 2 trên ta thu được   1 Q ≤ 5 1 + 2 (a2 + d 2 ) + 5(k2 + 2)(b2 + c2 ). k Do k = q√ 5−1 2 nên 1 + k12 = k2 + 2 = √ 3+ 5 2 , −a. Vì đẳng thức có thể xảy ra nên Lại có theo giả thiết thì a2 + b2 + c2 + d 2 ≥ 12 , nên từ Q = 25(x2 + y2 + z2 + t 2 ) ≥ vì thế ta có √ √ 15 + 5 5 2 15 + 5 5 Q≤ (a + d 2 + b2 + c2 ) ≤ . 2 2  a 2 = −b = c = − kd2 k2 Đẳng thức xảy ra khi và chỉ khi tức là a = ± √ k 4 , b = − ka2 , c = 2 2(k +1) a + b2 + c2 + d 2 = 1 √ 15+5 5 2 √ 7+3 5 2 . ( Đẳng thức xảy ra khi và chỉ khi a ,d k2 = cũng chính là giá tị lớn nhất của Q. x = y = kz2 = kt2 25(x2 + y2 + z2 + t 2 ) = 1 √ . 7+3 5 1√ 7+3 5 tức là x = y = ± q 1 √ (14+6 5)(k4 +1) và z = t = k2 x = k2 y, từ đây ta dễ dàng tìm được a, b, c, d thỏa mãn đẳng thức xảy ra. Và cũng vì đẳng thức có thể xảy ra nên 7+31√5 cũng chính là giá trị nhỏ nhất của Q. Bài toán được giải quyết hoàn toàn. 5 Bài O26. Cho n ≥ 3 là một số nguyên cho trước và xi > 1 (1 ≤ i ≤ n) là các số thực thỏa mãn  a − 2b + c = −5x    b − 2c + d = −5y Bây giờ, ta sẽ đi tìm giá trị nhỏ nhất của Q. Để tiến hành, ta sẽ đặt , khi đó ta b − 2a = −5z    c − 2d = −5t  a = 3x + 2y + 4z + t    b = 6x + 4y + 3z + 2t có . Đến đây, áp dụng bất đẳng thức Cauchy Schwarz một lần nữa, ta có c = 4x + 6y + 2z + 3t    d = 2x + 3y + z + 4t www.mathvn.com n xi2 ≥ S = ∑ xi xi − 1 i=1 ∀i = 1, 2, . . . , n. Tìm giá trị lớn nhất có thể có của S.1 (Olympic toán Romania 2008) 1 Trong đề bài gốc, bài toán được cho giả thiết là n ≥ 2 và yêu cầu tìm sup S, nhưng xét thấy với n = 2 thì rõ ràng sup S = +∞ nên chúng tôi đã sửa lại thành như trên. www.mathvn.com Những bài bất đẳng thức từ các cuộc thi giải toán 27 Lời giải (V. Q. B. Cẩn). Từ giả thiết, ta dễ thấy Những bài bất đẳng thức tự sáng tạo và sưu tầm n xi2 ≥ ∑ xi = xi + ∑ xk > xi + (n − 1) ≥ xi + 2, xi − 1 i=1 k6=i suy ra xi2 > (xi − 1)(xi + 2), hay là 1 < xi < 2 với mọi i = 1, 2, . . . , n. Để giải bài toán này, ta cần xét √ S− S2 −4S . 2 trường hợp sau: S > 4, khi đó ta sẽ chứng minh rằng xi ≤ Thật vậy, bất đẳng thức ≥S có thể được viết dưới dạng tương đương là f (xi ) = xi2 − Sxi + S ≥ 0. Ta thấy√rằng f (xi ) là một tam √ 2 2 thức bậc hai của xi với hệ số cao nhất dương và có hai nghiệm phân biệt là S− S2 −4S và S+ S2 −4S . Vì √ √ 2 2 vậy, để f (xi ) ≥ 0, ta cần có xi ≤ S− S2 −4S hoặc xi ≥ S+ S2 −4S . Tuy nhiên, khả năng thứ hai là không thể xảy ra, bởi vì nếu nó xảy ra ta sẽ có xi > S2 , mà xi < 2 nên ta thu được S < 4, điều này mâu thuẫn với giả thiết mà ta đang xét, đó là S > 4. Như vậy, ta phải có √ S − S2 − 4S xi ≤ ∀i = 1, 2, . . . , n. 2 Từ đó, ta suy ra được √ S = x1 + x2 + · · · + xn ≤ n · S ≤ 4, khi đó ta dễ dàng kiểm tra được S ≤ 4 ≤ n2 n−1 , S≤ n2 n−1 . (Võ Quốc Bá Cẩn) Lời giải (V. Q. B. Cẩn). Nhân cả hai vế của bất đẳng thức đã cho với a2 b2 + b2 c2 + c2 a2 > 0, ta có thể viết lại nó dưới dạng  3 ∑ a2 b2  bc(a2 b2 + b2 c2 + c2 a2 ) cyc + − ∑bc ≥ 0, bc − ∑ a4 + 2b2 c2 ∑ a2 cyc cyc cyc bc(a2 − b2 )(a2 − c2 ) + a4 + 2b2 c2 cyc Trong trường hợp thứ hai, ∑ nên trong mọi trường hợp ta đều có 3 ∑ a2 b2 − ∑ ab(a2 + b2 ) − ∑ a2 bc cyc cyc cyc ∑ a2 ≥ 0, cyc hay là n2 . n−1 Đẳng thức xảy ra khi và chỉ khi x1 = x2 = · · · = xn = bc ca ab 3 + + ≤ . a4 + 2b2 c2 b4 + 2c2 a2 c4 + 2a2 b2 a2 + b2 + c2 tương đương S − S2 − 4S . 2 Đến đây, bằng một chút biến đổi đơn giản, ta dễ dàng thu được S ≤ Bài CH1. Giả sử a, b, c là độ dài ba cạnh của một tam giác, hãy chứng minh bất đẳng thức sau xi2 xi −1  ∑ 2ab + 2ac − b2 − c2 (a − b)(a − c) bc(a2 − b2 )(a2 − c2 ) cyc − ≥ 0. 2∑ a4 + 2b2 c2 ∑ a2 cyc n n−1 . cyc Bất đẳng thức cuối này có dạng X(a − b)(a − c) +Y (b − c)(b − a) + Z(c − a)(c − b) ≥ 0, trong đó X= 2bc(a + b)(a + c)(a2 + b2 + c2 ) + b2 + c2 − 2a(b + c) + (b − c)2 , a4 + 2b2 c2 và các biểu thức Y, Z tương tự. Đây là một dạng của bất đẳng thức Vornicu Schur nên ta nghĩ ngay đến việc sử dụng bất đẳng thức này để giải bài toán đã cho. Muốn như vậy, yêu cầu đầu tiên ta cần phải thỏa mãn đó là X,Y, Z là những đại lượng không âm, và may mắn thay, điều này luôn đúng. Thật vậy, bất đẳng thức X ≥ 0 (các bất đẳng thức Y ≥ 0 và Z ≥ 0 được xét tương tự) tương đương với 2bc(a + b)(a + c)(a2 + b2 + c2 ) + b2 + c2 − 2a(b + c) + (b − c)2 ≥ 0. a4 + 2b2 c2 Theo bất đẳng thức AM – GM thì (a + b)(a + c)(a2 + b2 + c2 ) (a2 + bc)(a2 + 2bc) > > 1. a4 + 2b2 c2 a4 + 2b2 c2 Vì thế, ta có X > 2bc + b2 + c2 − 2a(b + c) = (b + c)(b + c − 2a), dẫn đến kết luận của ta là hiển nhiên nếu b + c ≥ 2a. Xét trường hợp a ≥ t = b+c 2 , ta sẽ chứng minh rằng (a + b)(a + c)(a2 + b2 + c2 ) ≥ 3a4 + 6b2 c2 . www.mathvn.com (∗) www.mathvn.com Những bài bất đẳng thức tự sáng tạo và sưu tầm 29 Khi đó, ta sẽ có X ≥ 6bc + b2 + c2 − 2a(b + c) + (b − c)2 = 2(b + c)(b + c − a) ≥ 0, chính là điều mà ta đang tìm cách chứng minh. Đặt x = bc ≤ t 2 thì bất đẳng thức (∗) có thể được viết lại thành (a2 + 2ta + x)(a2 + 4t 2 − 2x) ≥ 3a4 + 6x2 , hay là 30 tương đương √ √ √ 9 abc(a + b)(b + c)(c + a) c(a + b) − 2 abc ≥ . 2(a + b + c)(ab + bc + ca) cyc √ √ 2 √ √ √ Đến đây, ta hãy để ý rằng ∑ c(a + b) − 6 abc = ∑ c a − b và ∑ cyc    −8x2 + 4t 2 − 4at − a2 x + a2 + 4t 2 a2 + 2ta − 3a4 ≥ 0. Do 4t 2 − 4at − a2   a2 + 2ta − 3a4 = (2t − a)(2a3 + 2a2t + at 2 − 2t 3 ) ≥ 0. Như vậy, khẳng định trên của ta đã được chứng minh. Bây giờ, giả sử a ≥ b ≥ c, ta có X(a − b)(a − ≥ 0 nên c) ≥ 0 và a − c − bc (a − b) = (b−c)(b+c−a) c ∑ X(a − b)(a − c) ≥ Y (b − c)(b − a) + Z(a − c)(b − c) suy ra bất đẳng thức trên là hệ quả của bất đẳng thức sau √ √ 2 √ √ c abc(a − b)2 c a− b ≥ , 2(a + b + c)(ab + bc + ca) hay là cyc b (bZ − cY )(a − b)(b − c) ≥ Y (b − c)(b − a) + Z · (a − b) · (b − c) = . c c Vì thế, ta chỉ cần chứng minh bZ ≥ cY là bài toán được giải quyết xong, điều này tương đương với việc chứng minh  2   c2 (a + b) b (a + c) 2a(a2 + b2 + c2 )(b + c) − 2 2 + 2 (b − c) a2 + b2 + c2 − ab − ac ≥ 0. 2 2 4 4 2a b + c 2a c + b b2 (a+c) 2a2 b2 +c4 cyc √ √ abc ∑ c(a − b)2 √ 9 abc(a + b)(b + c)(c + a) cyc − 4 abc = , 2(a + b + c)(ab + bc + ca) 2(a + b + c)(ab + bc + ca) < 0 nên V T ≥ −8t 4 + (4t 2 − 4at − a2 )t 2 + a2 + 4t 2 Let the solutions say your method - Võ Quốc Bá Cẩn √ 2 √ √ a+ b . 2(a + b + c)(ab + bc + ca) ≥ c ab √ √ 2 Đây là một bất đẳng thức đúng bởi vì 2(a + b + c) ≥ 2(a + b) ≥ a + b (theo Cauchy Schwarz) √ và ab + bc + ca ≥ bc + ca ≥ 2c ab (theo AM – GM). Và như vậy, bài toán đã cho đã được chứng minh xong. Dễ thấy đẳng thức xảy ra khi và chỉ khi a = b = c hoặc a = 0 hoặc b = 0 hoặc c = 0. Bài CH3. Cho các số thực dương a, b thỏa mãn a + b = a4 + b4 . Chứng minh rằng c2 (a+b) 2a2 c2 +b4 Dễ dàng đánh giá được ≥ nên bất đẳng thức này hiển nhiên đúng. Bài toán được chứng minh xong. Đẳng thức xảy ra khi và chỉ khi a = b = c hoặc a = 2b = 2c và các hoán vị tương ứng. Bài CH2. Cho các số thực không âm a, b, c thỏa mãn không có hai số nào đồng thời bằng 0. Chứng minh bất đẳng thức sau s s r 9abc ab bc ca + + ≥ 1+ . (a + c)(b + c) (b + a)(c + a) (c + b)(a + b) 2(a + b + c)(ab + bc + ca) (Dương đức Lâm) Lời giải (V. Q. B. Cẩn). Sử dụng các bất đẳng thức Cauchy Schwarz và AM – GM, ta có  √ √ p ab ab + c ab(a + c)(b + c) VT = ∑ ≥∑ (a + c)(b + c) cyc cyc (a + c)(b + c) √ √ ab c =∑ + abc ∑ cyc (a + c)(b + c) cyc (a + c)(b + c) √ √ c 2abc = 1 + abc ∑ − . (a + c)(b + c) (a + b)(b + c)(c + a) cyc Do đó, để chứng minh bất đẳng thức đã cho, ta chỉ cần chứng minh được √ √ c 2abc 9abc abc ∑ − ≥ , (a + b)(b + c)(c + a) 2(a + b + c)(ab + bc + ca) cyc (a + c)(b + c) www.mathvn.com 3 3 aa bb ≤ 1 ≤ aa bb . (Vasile Cirtoaje) Lời giải (V. Q. B. Cẩn). Trước hết, ta sẽ chứng minh bất đẳng thức bên trái. Dễ thấy rằng nó tương đương với a ln a + b ln b ≤ 0. Sử dụng bất đẳng thức quen thuộc ln x ≤ x − 1 ∀x > 0, ta có 3a ln a − (a4 − a) ≤ 3a(a − 1) − (a4 − a) = −a(a + 2)(a − 1)2 ≤ 0, từ đó suy ra 3(a ln a + b ln b) ≤ (a4 − a) + (b4 − b) = 0. Và như thế, bất đẳng thức trên trái đã được chứng minh xong. Bây giờ, ta sẽ chứng minh bất đẳng thức bên phải. Cũng tương tự như trên, ta sẽ lấy logarith nepe hai vế và viết lại bất đẳng thức dưới 4 dạng a3 ln a + b3 ln b ≥ 0. Xét hàm số sau với x ∈ (0, 2) : f (x) = 3 ln x − x x−x 3 , ta có f 0 (x) = 3 2 (x − 1)(2 + 2x − x2 ) −1− 3 = . x x x3 Suy ra phương trình f 0 (x) = 0 chỉ có một nghiệm duy nhất trên khoảng (0, 2) là x = 1. Mặt khác, qua 1 thì f 0 (x) đổi dấu từ âm sang dương nên ta tìm được f (x) ≥ f (1) = 0 với mọi x ∈ (0, 2). Đến đây, 4 sử dụng giả thiết của bài toán và bất đẳng thức trung bình lũy thừa, ta có a + b = a4 + b4 ≥ (a+b) 8 , suy ra a + b ≤ 2, mà a, b là các số dương nên a, b ∈ (0, 2). Vì thế, áp dụng bất đẳng thức vừa chứng minh, ta có b4 − b a4 − a 3(a3 ln a + b3 ln b) ≥ a3 · + b3 · = a4 + b4 − a − b = 0. a3 b3 www.mathvn.com Những bài bất đẳng thức tự sáng tạo và sưu tầm 31 Bất đẳng thức bên phải được chứng minh xong. Dễ thấy ở cả hai bất đẳng thức (bên trái và bên phải) đẳng thức chỉ xảy ra tại một điểm là (a, b) = (1, 1). Bài CH4. Chứng minh rằng với mọi số thực không âm a, b, c thỏa mãn không có hai số nào trong chúng có thể đồng thời bằng 0, bất đẳng thức sau luôn được thỏa mãn ≥ cyc 9 a2 +b2 +c2 +3ab+3bc+3ca nên bất đẳng thức b2 b2 +3ca và a2 + b2 + c2 + 3(ab + bc + ca) ≥ 12bc, suy ra V T −V P ≥ (a2 − b2 ) − 2 2 9a · a +b +c +3(ab+bc+ca) · (a − b) (a − b)(a + 4b) 12 = ≥ 0. a2 + b2 + c2 + 3(ab + bc + ca) 4 Đến đây, với để ý rằng Z(c − a)(c − b) ≥ 0 và a − c ≥ ab (b − c) ≥ 0, ta thu được a (aX − bY )(a − b)(b − c) ≥ X(a − b) · (b − c) +Y (b − c)(b − a) = ≥ 0. b b Bài toán được giải quyết xong. Đẳng thức xảy ra khi và chỉ khi a = b = c hoặc (a, b, c) là một hoán vị của bộ số (t,t, 0) với t là một số dương bất kì. a(a − b)(a − c) 9abc (a + b + c)3 + 2 + − (a + b + c) ≥ 0, a2 + 3bc a + b2 + c2 + 3ab + 3bc + 3ca 4(ab + bc + ca) Bài CH5. Cho các số thực dương a, b, c thỏa mãn ab + bc + ca + abc = 4. Tìm tất cả các số thực k sao cho bất đẳng thức sau luôn được thỏa mãn (k + bc)(k + ca)(k + ab) ≥ (k + 1)3 . tương đương (Vuonga2khtn*2 ) 2 Lời giải (V. Q. B. Cẩn). Cho c = t > 0 và a = b = t+1 thì ta dễ thấy ab + bc + ca + abc = 4 và bất i  h 2t 2 4 đẳng thức đã cho trở thành k + t+1 k + (t+1) ≥ (k + 1)3 , tương đương 2 9abc 9abc a(a − b)(a − c) (a + b + c)3 + 9abc + − (a + b + c) ≥ − . a2 + 3bc 4(ab + bc + ca) 4 ∑ ab ∑ a2 + 3 ∑ ab cyc ∑ cyc Ta có ≥ cyc trên được suy ra từ ∑ a2 a2 +3bc ∑ X(a − b)(a − c) ≥ X(a − b)(a − c) +Y (b − c)(b − a) a(a − b)(a − c) 1 (a + b + c)3 + abc ∑ 2 + − (a + b + c) ≥ 0. a2 + 3bc 4(ab + bc + ca) cyc cyc a + 3bc ∑ cyc và các biểu thức Y, Z tương tự. Bây giờ, giả sử rằng a ≥ b ≥ c, ta sẽ chứng minh aX ≥ bY, tương đương   a2 b2 9abc(a − b) (a2 − b2 ) + 4(ab + bc + ca) 2 − 2 ≥ 2 . a + 3bc b + 3ca a + b2 + c2 + 3(ab + bc + ca) 2 (Dương đức Lâm) Lời giải (V. Q. B. Cẩn). Nhân cả hai vế của bất đẳng thức cho ab + bc + ca > 0, và để ý rằng a − a(ab+bc+ca) = a(a−b)(a−c) + a2abc , ta có thể viết lại nó như sau a2 +3bc a2 +3bc +3bc Áp dụng bất đẳng thức Cauchy Schwarz, ta có ∑ Let the solutions say your method - Võ Quốc Bá Cẩn Một điều dễ thấy là b c (a + b + c)3 a + 2 + 2 ≤ . 2 a + 3bc b + 3ca c + 3ab 4(ab + bc + ca)2 1 a2 +3bc 32 (a + b + c)3 + 9abc − 4(a + b + c)(ab + bc + ca) cyc cyc = ∑ a(a − b)(a − c) và (t − 1)2 [(k2 + k − 1)t 2 + (2k2 − 2k − 6)t + k2 − 3k − 1] ≥ 0. (t + 1)4 cyc a2 + b2 + c2 − (ab + bc + ca) = ∑(a − b)(a − c), Và như vậy, theo yêu cầu của đề bài, ta cần có (k2 + k − 1)t 2 + (2k2 − 2k − 6)t + k2 − 3k − 1 ≥ 0. Vế trái của bất đẳng thức này là một tam thức bậc 2 của t, và chúng ta đều biết rằng để nó không âm với mọi t dương thì một điều kiện cần là hai hệ số cao nhất phải không âm, tức là √ và thấp nhất √ k2 + k − 1 ≥ 0 và k2 − 3k − 1 ≥ 0. Từ đây, ta tìm được k ≤ − 1+2 5 hoặc k ≥ 3+2 13 . Ta sẽ chứng minh đây chính là tập hợp tất cả các giá trị cần tìm của k, tức là cyc nên bất đẳng thức trên tương đương với 9abc ∑ (a − b)(a − c) ∑ a(a − b)(a − c) a(a − b)(a − c) cyc cyc   . + ≥ a2 + 3bc 4(ab + bc + ca) cyc 4 ∑ ab ∑ a2 + 3 ∑ ab ∑ cyc cyc (k + bc)(k + ca)(k + ab) ≥ (k + 1)3 . cyc Để chứng minh, √ chúng ta sẽ chia làm 2 trường hợp + Xét k ≥ 3+2 13 . Khi đó, áp dụng kết quả bài O2, ta có Ta thấy bất đẳng thức này có dạng X(a − b)(a − c) +Y (b − c)(b − a) + Z(c − a)(c − b) ≥ 0, với 4a(ab + bc + ca) 9abc − 2 a2 + 3bc a + b2 + c2 + 3(ab + bc + ca) 4a(ab + bc + ca) 9abc ≥ a+ 2 − a + b2 + c2 + 3(ab + bc + ca) a2 + b2 + c2 + 3(ab + bc + ca) a[a2 + 7a(b + c) + (b − c)2 ] = 2 ≥ 0, a + b2 + c2 + 3(ab + bc + ca) (k + bc)(k + ca)(k + ab) = k3 + k2 (ab + bc + ca) + kabc(a + b + c) + a2 b2 c2 X = a+ ≥ k3 + k2 (ab + bc + ca) + kabc(ab + bc + ca) + a2 b2 c2 = k3 + k2 (4 − abc) + kabc(4 − abc) + a2 b2 c2 . 2 Những bài mà chúng tôi không biết tên thật của tác giả và chỉ biết nickname, chúng tôi sẽ ghi nickname kèm theo dấu * ở phía sau. Khi nào biết được tên thật sự của tác giả, chúng tôi xin sửa lại và ghi đúng tên của người đặt ra bài toán. www.mathvn.com www.mathvn.com Những bài bất đẳng thức tự sáng tạo và sưu tầm 33 Mà k3 + k2 (4 − abc) + kabc(4 − abc) + a2 b2 c2 − (k + 1)3 = (1 − abc)[(k − 1)abc + k2 − 3k − 1] ≥ 0 (do 1 ≥ abc (đánh giá này được suy ra trực tiếp từ giả thiết), (k − 1)abc ≥ 0 và k2 − 3k − 1 ≥ 0) nên hiển nhiên (k + bc)(k + ca)(k + ab) ≥ (k + 1)3 . √ √ √ √ 1+ 5 + Xét k ≤ − 2 < −1. Đặt ab = x, bc = y, ca = z thì ta có x2 + y2 + z2 + xyz = 4, và ta phải 2 2 2 3 chứng minh (k + x )(k + y )(k + z ) ≥ (k + 1) . Áp dụng bất đẳng thức Schur bậc 3, ta có 4(x2 y2 + y2 z2 + z2 x2 ) ≤ = 9x2 y2 z2 x 2 + y2 + z2 + (x2 + y2 + z2 )2 34 Let the solutions say your method - Võ Quốc Bá Cẩn  a+b Từ đây, ta thấy nếu 6(a + b) + 1 − 16(1 − c)(1 − d) ≥ 0 thì P(a, b, c, d) ≥ P a+b 2 , 2 , c, d . Nếu 6(a + b) + 1 − 16(1 − c)(1 − d) ≤ 0 và a + b ≤ 1 thì P(a, b, c, d) ≥ P(a + b, 0, c, d). Nếu 6(a + b) + 1 − 16(1 − c)(1 − d) ≤ 0 và a + b ≥ 1 thì P(a, b, c, d) ≥ P(a + b − 1, 1, c, d). Những lập luận này chứng tỏ rằng, để chứng minh bất đẳng thức đã cho, ta chỉ cần chứng minh nó đúng trong ba trường hợp sau là đủ a = b, ab = 0 và (1 − a)(1 − b) = 0. Hoàn toàn tương tự, ta cũng thấy rằng chỉ cần chứng minh bất đẳng thức đúng trong ba trường hợp c = d, cd = 0 và (1 − c)(1 − d) = 0 thì bài toán cũng được giải quyết xong. Kết hợp hai lập luận này lại và loại bớt những trường hợp trùng nhau, ta có thể đưa bài toán về xét trong 4 trường hợp sau + Xét a = b và c = d. Khi đó, bất đẳng thức cần chứng minh trở thành 9t 2 + (4 − t)2 (t = xyz ≤ 1). 4−t 2a3 + 2c3 + Từ đó suy ra   2 Đặt t = a + c và x = ac 0 ≤ x ≤ t4 , ta có thể viết lại nó như sau (k + x2 )(k + y2 )(k + z2 ) − (k + 1)3 = k2 (x2 + y2 + z2 − 3) + k(x2 y2 + y2 z2 + z2 x2 − 3) + t 2 − 1 " 2 # 9t + (4 − t)2 ≥ k2 (1 − t) + k 4−t − 3 + t2 − 1 4   2 − 20t + 16) k(t = (1 − t) k2 + −t −1 . 4(4 − t) 2t(t 2 − 3x) + f (x) = 32x2 + 4 (15 − 22t) x + 8t 3 + 34t 2 − 64t + 27 ≥ 0. f 0 (x) = 64x +4(15 −22t) ≤ 16t 2 +4(15 −22t) = 4(4t 2 −22t +15) ≤ 2 0, dẫn đến f (x) là hàm giảm với mọi x ≤ t4 , và ta thu được  2 t f (x) ≥ f = (2t 2 − 10t + 27)(t − 1)2 ≥ 0. 4 Nếu 4t 2 −22t +15 ≤ 0 thì ta có k(t 2 − 20t + 16) t[(k + 4)t − 16(k + 1)] t[(k + 4)t − 16(k + 1)t] 3t 2 (5k + 4) −t −k = ≥ =− ≥ 0, 4(4 − t) 4(4 − t) 4(4 − t) 4(4 − t) nên k(t 2 − 20t + 16) k2 + − t − 1 ≥ k2 + k − 1 ≥ 0. 4(4 − t) Như vậy, khẳng định của ta đãđược chứng minh xong. Và do  đó, tập hợp tất cả các giá trị của k thỏa √ i h √ mãn yêu cầu của đề bài là k ∈ −∞, − 1+2 5 ∪ 3+2 13 , +∞ . Nếu 15 − 22t ≥ 0 thì bất đẳng thức là hiển nhiên đúng bởi vì ta luôn có 8t 3 + 34t 2 − 64t + 27 > 0 với mọi t ≥ 0. √ 11− 61 Nếu 15 − 22t ≤ 0 và 4t 2 − 22t + 15 ≥ 0 thì ta có 15 , khi đó dễ thấy 22 ≤ t ≤ 4 Bài CH6. Cho a, b, c, d là các số thực không âm thỏa mãn a2 + b2 + c2 + d 2 = 1. Chứng minh rằng ∆0f = 4(15 − 22t)2 − 32(8t 3 + 34t 2 − 64t + 27) = −4(64t 3 − 212t 2 + 148t − 9) < 0, a3 + b3 + c3 + d 3 + 8(1 − a)(1 − b)(1 − c)(1 − d) ≥ 1. (Phạm Văn Thuận) Lời giải (V. Q. B. Cẩn). Chúng tôi xin được giới thiệu cùng bạn đọc chứng minh sau. Mặc dù là một chứng minh không đẹp nhưng nó lại là một ý tưởng mới về bất đẳng thức (chuyển từ bất đẳng thức thuần nhất sang dạng không thuần nhất). Từ giả thiết, ta dễ dàng suy ra được a, b, c, d ∈ [0, 1] và ta cũng có thể viết lại bất đẳng thức cần chứng minh dưới dạng P(a, b, c, d) ≥ 0, trong đó cyc t 2 − 2x 5 + 8(1 − t + x)2 − ≥ 0, 2 4 tương đương Lại có P(a, b, c, d) = ∑ a3 + a2 + c2 5 + 8(1 − a)2 (1 − c)2 − ≥ 0. 2 4 1 5 a2 + 8(1 − a)(1 − b)(1 − c)(1 − d) − . 4∑ 4 cyc nên f (x) luôn đạt giá trị không âm với mọi a, c ∈ [0, 1]. Trường hợp thứ nhất được giải quyết xong. + Xét a = b và d = 0. Khi đó, bất đẳng thức cần chứng minh trở thành 2a3 + c3 + tương đương 2a3 + Bây giờ, ta sẽ chứng minh bất đẳng thức P(a, b, c, d) ≥ 0 đúng với mọi a, b, c, d ∈ [0, 1] mà không cần thiết phải thỏa mãn a2 + b2 + c2 + d 2 = 1. Thật vậy, ta có   a+b a+b (a − b)2 [6(a + b) + 1 − 16(1 − c)(1 − d)] P(a, b, c, d) − P , , c, d = , 2 2 8 ab[6(a + b) + 1 − 16(1 − c)(1 − d)] P(a, b, c, d) − P(a + b, 0, c, d) = − , 2 (1 − a)(1 − b)[6(a + b) + 1 − 16(1 − c)(1 − d)] P(a, b, c, d) − P(a + b − 1, 1, c, d) = − . 2 www.mathvn.com 2a2 + c2 5 + 8(1 − a)2 (1 − c) − ≥ 0, 4 4 a2 (1 − c)(27 − 5c − 4c2 ) − 8a(2 − a)(1 − c) + ≥ 0. 2 4 Do 1 − c ≥ 0 và 27 − 5c − 4c2 ≥ 27 − 9c nên ta chỉ cần chứng minh được 2a3 + a2 9(1 − c)(3 − c) − 8a(2 − a)(1 − c) + ≥ 0, 2 4 hay là f (c) = 9c2 − 4(8a2 − 16a + 9)c + 8a3 + 34a2 − 64a + 27 ≥ 0. Nếu 16a2 −32a+9 ≥ 0 thì ta có f 0 (c) = 18c−4(8a2 −16a+9) ≤ 18−4(8a2 −16a+9) = −2(16a2 − 32a + 9) ≤ 0 nên f (c) là hàm giảm với mọi c ≤ 1, và ta thu được f (c) ≥ f (1) = 2a2 (4a + 1) ≥ 0. www.mathvn.com Những bài bất đẳng thức tự sáng tạo và sưu tầm Nếu 16a2 − 32a + 9 ≤ 0 thì √ 4− 7 4 35 36 Let the solutions say your method - Võ Quốc Bá Cẩn Bất đẳng thức thứ nhất tương đương với ≤ a ≤ 1, khi đó bằng cách tính biệt thức của f (c), ta dễ thấy ∆0f = 4(8a2 − 16a + 9)2 − 9(8a3 + 34a2 − 64a + 27) = 256a4 − 1096a3 + 1294a2 − 576a + 81 < 0, dẫn đến f (c) ≥ 0 và trường hợp thứ hai cũng được giải quyết xong. 2 2 2 + Xét b = 1. Khi đó, bất đẳng thức cần chứng minh trở thành a3 + c3 + d 3 + a +c4 +d ≥ 0, là một bất đẳng thức hiển nhiên đúng do a, c, d là những số không âm. + Xét b = 0 và d = 0. Khi đó, ta phải chứng minh a3 + c3 + 2 2 2 b2 c2 a2 a b c b c a + + + + + ≥ + + + 3, a2 b2 c2 b c a a b c  b c a 2 ≥ ba + bc + ac và ab + bc + ac ≥ 3 nên bất đẳng thức này hiển nhiển a+b+c mà ab2 + bc2 + ac2 ≥ 13 đúng. Xét bất đẳng thức thứ hai, lấy căn bậc hai hai vế, ta thấy rằng bất đẳng thức này tương đương với a2 + c2 5 + 8(1 − a)(1 − c) − ≥ 0. 4 4 b c a + + ≥ a + b + c. a b c   2 Đặt t = a + c và x = ac 0 ≤ x ≤ t4 , bất đẳng thức này trở thành t(t 2 − 3x) + Từ giả thiết, áp dụng các bất đẳng thức Chebyshev và AM – GM, ta có √ 3 3(a + b + c) = 3(a3 + b3 + c3 ) ≥ (a + b + c)(a2 + b2 + c2 ) ≥ 3 a2 b2 c2 (a + b + c), 5 t 2 − 2x + 8(1 − t + x) − ≥ 0, 4 4 suy ra 1 ≥ abc, và ta thu được hay là 2 6(5 − 2t)x + (1 − t)(27 − 5t − 4t ) ≥ 0. Nếu t ≤ 1 thì bất đăng thức cuối là hiển nhiên bởi vì ta có 6(5 − 2t)x ≥ 0 và (1 −t)(27 − 5t − 4t 2 ) ≥ 0. Trong trường hợp ngược lại, sử dụng đánh giá (1 − a)(1 − c) ≥ 0, ta suy ra được x ≥ t − 1, dẫn đến 6(5 − 2t)x + (1 − t)(27 − 5t − 4t 2 ) ≥ 6(5 − 2t)(t − 1) + (1 − t)(27 − 5t − 4t 2 ) = (4t − 3)(t − 1)2 ≥ 0.       b c a 1 2b a 1 2a c 1 2c b + + = + + + + + a b c 3 a c 3 c b 3 b a r r r 2 2 2 a+b+c 3 b 3 a 3 c ≥ + + = √ ≥ a + b + c. 3 ac bc ab abc Bài toán được chứng minh xong. Dễ thấy đẳng thức xảy ra khi và chỉ khi a = b = c = 1. Trường hợp thứ tư cũng được giải quyết xong. Và như thế, phép chứng minh của ta cũng được hoàn tất. Dễ thấy đẳng thức xảy ra khi và chỉ khi a = b = c = d = 21 hoặc a = 1, b = c = d = 0 và các hoán vị tương ứng. Bài CH8. Cho các số thực không âm a, b, c thỏa mãn Bài CH7. Cho các số thực dương a, b, c thỏa mãn a + b + c = a3 + b3 + c3 . Chứng minh bất đẳng thức sau  2  c 2  a 2 a b c a+b+c b · + 2 · + 2 · ≥ . 2 a +1 b b +1 c c +1 a 2 Chứng minh bất đẳng thức sau 1 1 1 1 + + = . a2 + 47 b2 + 47 c2 + 47 24 r a + b + c ≥ 10 (Gabriel Dospinescu) (Yongyao*) 1 1 1 1 , b2 +47 , c2 +47 ≤ 47 nên từ giả thiết, ta có thể đặt được Lời giải (V. Q. B. Cẩn). Để ý rằng a2 +47 1−y 1 1 1 1−z = 1−x 47 , b2 +47 = 47 , c2 +47 = 47 với x, y, z là các số thực không âm nằm trong đoạn [0, 1]. Từ a2 +47 phép đặt này, chúng ta có thể dễ dàng suy ra được x + y + z = 25 24 , và ta phải chứng minh Lời giải (V. Q. B. Cẩn). Áp dụng bất đẳng thức Cauchy Schwarz, ta dễ thấy  2    a + 1 b2 + 1 c2 + 1 c a b 2 VT · + + ≥ + + . a b c b c a Do đó, ta chỉ cần chứng minh được     1 1 1 c a b 2 + + ≥ (a + b + c) a + b + c + + + . 2 b c a a b c r  c a b + + b c a 2 x + 1−x r y + 1−y r z 10 ≥√ . 1−z 23 Không mất tính tổng quát, ta có thể giả sử z = min{x, y, z}. Khi đó, dễ thấy x + y ≥ 32 (x + y + z) > 23 , dẫn đến (x + y)2 (x + y)3 (3x + 3y − 2)(x − y)2 + =− ≤ 0. x2 + y2 − (x3 + y3 ) − 2 4 4 Ta thấy rằng bất đẳng thức này chính là tổng của hai bất đẳng thức sau     c a b 2 1 1 1 + + ≥ (a + b + c) + + , b c a a b c và 47 . 23 Từ đây, áp dụng bất đẳng thức Holder, ta thu được s s r r r x y (x + y)3 (x + y)3 25 − 24z + ≥ ≥ = 2 . 3 (x+y)2 1−x 1−y x2 (1 − x) + y2 (1 − y) 23 + 24z − (x+y) ≥ (a + b + c)2 . 2 www.mathvn.com 4 www.mathvn.com Những bài bất đẳng thức tự sáng tạo và sưu tầm 37 Vì thế, để chứng minh bất đẳng thức trên, ta chỉ cần chứng minh được r r 25 − 24z z 10 + ≥√ . 2 23 + 24z 1−z 23 Nhận xét. Với cùng một cách làm như trên, chúng ta có thể chứng minh được một kết quả đẹp hơn rất nhiều là 1 1 1 + b2 +47 + c2 +47 a2 +47 1 = 24 Và một điều thú vị hơn nữa là bất đẳng thức này tương đương với kết quả sau (rất đẹp và khó) của tác giả Vasile Cirtoaje trên tạp chí Crux Mathematicorum ♣ Bài CH9. Cho các số thực không âm a, b, c thỏa mãn không có hai số nào đồng thời bằng 0 và tổng của chúng là 1. Tìm giá trị lớn nhất và giá trị nhỏ nhất của biểu thức sau b−c c−a a−b +√ +√ . P= √ c+a a+b b+c a= √ √ √ a + b, y = b + c và z = c + a, ta được z2 + x2 − y2 , 2 b= x2 + y2 − z2 , 2 c= √ √ √ √ √ a+b+ b+c+ c+a a+b a+b √ √ √ . = 1+ √ ≤ 1+ √ √ b+c+ c+a a+c+ b+c a+ b Những đánh giá này giúp ta thu được bất đẳng thức sau √  √ √ |ab(a − b)| a+ b+ a+b  √ |P| ≤ p √   √ √ √ √ ab(a + b) a + a + b b+ a+b a+ b √ √ a−b a−b √ √ 1 √ = √ + b − a = √ + b − a a+b a+b+c a+b √ √ √ √ a − b 1 a − b b − a = √ √ ≤√ + b − a + . a+b a+b a+b a+b Đặt x = Với mọi số thực không âm x, y, z thỏa mãn không có hai số nào đồng thời bằng 0 thì s s r 48 48y 48z 1+ + 1+ + 1+ ≥ 15. y+z z+x x+y Lời giải (V. Q. B. Cẩn). Đặt x = Let the solutions say your method - Võ Quốc Bá Cẩn và Việc chứng minh bất đẳng thức này khá đơn giản, xin được dành cho bạn đọc. Với mọi số thực không âm a, b, c thỏa mãn min{a, b, c} ≥ 1 và thì a + b + c ≥ 15. 38 b thì ta có a+b = 1 − x, và √ √ √ √ √ √ a−b b− a = x − (1 − x) + 1 − x − x = 2x − x + 1 − x − 1 = f (x). + √ a+b a+b a a+b 1 Tính đạo hàm f 0 (x), ta được f 0 (x) = 2 − 2√ − √1 . Giải phương trình f 0 (x) = 0, ta tìm được hai x √2 1−x √ √ √ 8+ 46−2 17 8− 46−2 17 và x = . Từ đó, bằng cách lập nghiệm của nó trong khoảng (0, 1) là x = 1 2 16 16  √  √ q √  √  √ 17 8+ 46−2 17 17 bảng biến thiên, dễ thấy rằng f 8− 46−2 ≤ f (x) ≤ f , hay là − 71−17 ≤ 16 16 32 q √ 17 f (x) ≤ 71−17 . Vì |P| ≤ | f (x)| nên ta cũng suy ra được 32 (Phạm Kim Hùng) s − y2 + z2 − x2 . 2 s √ √ 71 − 17 17 71 − 17 17 ≤P≤ . 32 32 √ √ √ √ √ √ √ (5− 17) 46−2 17 8− 46−2 17 17 , b = , c = 0 thì ta dễ thấy P = ; và cho Mặt khác, cho a = 8+ 46−2 16 16 32 √ √ √ √ √ √ √ (5− 17) 46−2 17 8− 46−2 17 8+ 46−2 17 a= ,b = , c = 0 thì P = − , nên ta đi đến kết luận 16 16 32 s s √ √ 71 − 17 17 71 − 17 17 max P = và min P = − . 32 32 Từ đó suy ra a − b (z2 + x2 − y2 ) − (x2 + y2 − z2 ) |P| = ∑ √ = ∑ cyc a + b cyc 2x z2 − y2 |(x − y)(y − z)(z − x)(x + y + z)| (x2 − y2 )(y2 − z2 )(z2 − x2 )(x + y + z) = = ∑ = cyc x xyz xyz(x + y)(y + z)(z + x)  √ √ √ |(a − b)(b − c)(c − a)| a + b + b + c + c + a =p  √  √ . √ √ √ √ (a + b)(b + c)(c + a) a + b + a + c b+c+ c+a b+c+ b+a Bài toán được giải quyết xong. Không mất tính tổng quát, giả sử rằng c = min {a, b, c} , ta có thể dễ dàng kiểm tra được các đánh giá sau |(a − b)(b − c)(c − a)| ≤ |ab(a − b)| , 1 1 p ≤p , (a + b)(b + c)(c + a) ab(a + b) 1 1 ,  √ ≤ √ √ √ √  √ √ √ a+b+ a+c b+c+ b+a a+ a+b b+ a+b Bài CH10. Chứng minh rằng với mọi số thực dương a, b, c, bất đẳng thức sau đây luôn được thỏa mãn a b c 3 + + ≤ . 2a2 + 3b + 2 2b2 + 3c + 2 2c2 + 3a + 2 7 (Phan Thành Nam) Lời giải (V. Q. B. Cẩn). Do tính hoán vị vòng quanh nên ta có thể giả sử b là số hạng nằm giữa a và c. Khi đó, có 2 trường hợp để xét là c ≥ b ≥ a và a ≥ b ≥ c. + Xét trường hợp c ≥ b ≥ a. Áp dụng bất đẳng thức AM – GM, ta có a a ≤ , 2a2 + 3b + 2 4a + 3b www.mathvn.com b b ≤ , 2b2 + 3c + 2 4b + 3c c c ≤ . 2c2 + 3a + 2 4c + 3a www.mathvn.com Những bài bất đẳng thức tự sáng tạo và sưu tầm 39 40 Vì thế, để chứng minh bất đẳng thức đã cho, ta chỉ cần chứng minh được Let the solutions say your method - Võ Quốc Bá Cẩn Mặt khác, dễ dàng kiểm tra được rằng 1 2b2 + 7b + 2 3 p + < , 2 − 9b + 26 26b 7 2 2(3b + 2) b c 3 a + + ≤ , 4a + 3b 4b + 3c 4c + 3a 7 do đó, kết hợp với trên, ta được g(c) < 37 . tức là 15(ab2 + bc2 + ca2 ) ≥ 8(a2 b + b2 c + c2 a) + 21abc. Nhưng bất đẳng thức này hiển nhiên đúng bởi vì ta có 8(ab2 + bc2 + ca2 ) ≥ 8(a2 b + b2 c + c2 a) (do c ≥ b ≥ a) và 7(ab2 + bc2 + ca2 ) ≥ 21abc (theo AM – GM). + Xét trường hợp a ≥ b ≥ c. Lúc này, có 2 khả năng xảy ra như sau • Khả năng thứ nhất a ≤ b + 3c. Với giả thiết này, thực hiện tương tự như trường hợp thứ nhất ở trên, ta thấy rằng bất đẳng thức của ta sẽ được chứng minh nếu ta có Bài toán được chứng minh xong. Đẳng thức xảy ra khi và chỉ khi a = b = c = 1. Bài CH11. Giả sử a, b, c là các số thực không âm thỏa mãn không có hai số nào đồng thời bằng 0 và tổng của chúng là 2. Chứng minh rằng khi đó, bất đẳng thức sau luôn được thỏa mãn r r r r a+b b+c c+a 2 + + ≥ 2 + . a2 + ab + b2 b2 + bc + c2 c2 + ca + a2 3 15(ab2 + bc2 + ca2 ) ≥ 8(a2 b + b2 c + c2 a) + 21abc, (Võ Quốc Bá Cẩn) Lời giải (V. Q. B. Cẩn, T. Q. Anh). Xét các số thực x, y, z sao cho x, y ≥ z ≥ 0, dễ thấy f (a) = (15c − 8b)a2 + (15b2 − 21bc − 8c2 )a + 15bc2 − 8b2 c ≥ 0. x+z x+z x+z 1   = ≥ = . x2 + xz + z2 x + 2z (x + z) x + 2z x + 2z (x + z) − 12 z(x − z) hay là Nếu 8b > 15c thì f (a) là một tam thức bậc 2 theo a với hệ số cao nhất âm, vì thế f (a) ≥ min { f (b), f (b + 3c)} . Mà f (b) = 7b(b − c)2 ≥ 0, f (b + 3c) = 7b3 − 17b2 c − 38bc2 + 111c3 > 0 nên hiển nhiên f (a) ≥ 0. Ngoài ra, ta cũng có đánh giá sau x+y x+y+z ≥ 2   2 . x2 + xy + y2 x + 2z + x + 2z y + 2z + y + 2z Trong trường hợp 15c ≥ 8b, tính đạo hàm f 0 (a), ta có Thật vậy, bất đẳng thức này tương đương với 2   2 x + 2z + x + 2z y + 2z + y + 2z x+y+z −1 ≥ − 1, 2 2 x + xy + y x+y f 0 (a) = 2a(15c − 8b) + 15b2 − 21bc − 8c2 ≥ 2b(15c − 8b) + 15b2 − 21bc − 8c2 = (8c − b)(b − c) ≥ 0, từ đó suy ra f (a) là hàm đồng biến, và ta suy ra f (a) ≥ f (b) ≥ 0. tương đương • Khả năng thứ hai a ≥ b + 3c. Trong khả năng này, sử dụng bất đẳng thức AM – GM, ta thu được các đánh giá a 1 ≤ p , 2a2 + 3b + 2 2 2(3b + 2) và c c c ≤ ≤ . 2c2 + 3a + 2 4c + 3a 3b + 13c Do đó, ta chỉ cần chứng minh bất đẳng thức sau đúng với mọi 0 < c ≤ b thì bài toán được giải quyết xong 1 b c 3 g(c) = p + + ≤ . 2 2(3b + 2) 2b2 + 3c + 2 3b + 13c 7 Ta sẽ chứng minh bất đẳng thức này đúng với mọi c > 0 mà không cần phải thỏa mãn c ≤ b. Thật vậy, ta có   1 1 g0 (c) = 3b − . (3b + 13c)2 (2b2 + 3c + 2)2 Phương trình g0 (c) = 0 chỉ có một nghiệm dương duy nhất là c0 = g0 (c) đổi dấu từ dương sang âm nên với mọi c > 0, ta có  g(c) ≤ g 2b2 − 3b + 2 10  2b2 −3b+2 10 > 0. Qua c0 thì 2b2 + 7b + 2 1 = p + . 2 2(3b + 2) 26b2 − 9b + 26 3z(2x + 2y + z) z ≥ (hiển nhiên đúng). 4(x2 + xy + y2 ) x + y Bây giờ, trở lại bài toán của ta. Do tính đối xứng nên ta có thể giả sử rằng a ≥ b ≥ c, khi đó từ hai đánh giá trên, ta được b+c 1 ≥ , b2 + bc + c2 b + 2c và c+a 1 ≥ , c2 + ca + a2 a + 2c a+b a+b+c ≥ 2   2 . a2 + ab + b2 a + 2c + a + 2c b + 2c + b + 2c Như vậy, ta chỉ cần chứng minh được 1 1 √ +√ + u v r 2 ≥ 2+ u2 + uv + v2 r 2 , 3 trong đó u = a + 2c và v = b + 2c . Đây là một bài tập rất tốt cho phép cân bằng hệ số trong việc sử dụng bất đẳng thức AM – GM. Xin được dành cho bạn đọc để hoàn thiện nốt chứng minh này. Chú ý rằng ở bất đẳng thức đã cho, đẳng thức xảy ra khi và chỉ khi a = b = 1, c = 0 và các hoán vị tương ứng. Nhận xét. Hoàn toàn tương tự, ta cũng có thể chứng minh được kết quả tổng quát hơn vẫn còn đúng www.mathvn.com www.mathvn.com
- Xem thêm -

Tài liệu liên quan